You are on page 1of 85

INTERNATIONAL INDIAN SCHOOL, JEDDAH

GRADE X
SUBJECT- ENGLISH
STUDY MATERIAL 2023-24

PREPARED BY:
READING SECTION:MRS. SHAGUFTA SHAFEEQ (GS)
GRAMMAMR &:MRS. ZOYA FAROOQUI (GS) AND MR. ABDUL HAKEEM (BS)
WRITING SECTION:MRS. GULISTAN (GS)
LITERATURE SECTION:MS. RAHAT (GS) &MRS. SHAGUFTA SHAFEEQ (GS)

1
Reading Section
Passage:1
Read the following text. 10 m
(1) As a high school student, studying poetry can be a rollercoaster ride. This journey is punctuated by
moments of profound appreciation for simpler pieces and intermittent frustration with more complex
works. Let's be real here - some poems are just plain confusing and no amount of re-reading seems to help
decipher the intended meaning. The puzzlement that results from such instances can be both vexing and de
motivating. If solving a riddle is what was intended, then playing Sudoku is a better option. One is led to
ponder if obscurity was the goal.

(2) Conversely, some pieces resonate with the reader's soul-stirring feelings of warmth, happiness, and
connection to the world. Often, these compositions centre on themes that are universally understood, such
as love, nature, or faith. Being able to actually understand what the poet is trying to say can feel like a little
victory and is a welcome relief after grappling with more perplexing poetry.

(3) Then there are poems that are emotionally charged; the ones that make the reader curl up in a ball and
cry or jump up and down with joy. One is left in awe of the poet's ability to convey emotion through words.
Let’s not forget the downright weird poems. These are the ones that defy categorization and leave the
reader to their own devices in attempting to interpret meaning. The author's use of figurative language and
unconventional imagery can create a sense of bewilderment that is either intriguing or off-putting.
Regardless, the reader can appreciate the uniqueness of the work.

(4) Despite the wide range of emotions and reactions that come with studying poetry, it can be a rewarding
pursuit. Not only does reading poetry allow one to appreciate the artistic beauty of the written word but also
enables one to develop crucial critical thinking and analytical skills. The process of unlocking a poem's
meaning can feel like cracking a code or solving a puzzle but the sense of accomplishment derived from
mastering a challenging piece can be deeply gratifying. Finally, impressing an English teacher with a well-
analysed poem can be a source of pride and validation.

(5) Overall, studying poetry is like a box of mixed chocolates, you never know what you're going to get. But
whether it's complex, emotional, simple, or just downright weird, there's always something to be gained from
the experience. So, let's applaud all the poets out there, for making us laugh, cry, scratch our heads, and
occasionally feel like a genius.

Answer the following questions, based on the passage above.


i. Which of the following statements best describes the author's attitude towards studying poetry?
A. Finds poetry to be a frustrating and meaningless endeavor.
B. Believes that the emotional rollercoaster of studying poetry is not worth the effort.
C. Recognizes the challenges of studying poetry but also acknowledges the rewards it offers.
D. Feels that poetry is too obscure and abstract for the average person to appreciate.

ii. What is the tone of the writer in the given lines from paragraph (1)? Rationalise your response in about 40
words.
If solving a riddle is what was intended, then playing Sudoku is a better option. One is led to ponder if
obscurity was the goal.

iii Complete the sentence appropriately.


The author's use of vivid imagery in the paragraph (3), such as "curl up in a ball and cry" and "jump up and
down with joy", greatly affects the reader because ______________.
iv The passage includes some words that are opposites of each other. From the sets (a)-(e) below, identify
two sets of antonyms:

(a) intriguing and off-putting (b) deciphering and interpreting


(c) appreciate and applaud (d) simple and challenging
(e) emotions and feelings

v Complete the sentence appropriately.


We can say that the author's tone becomes more neutral and objective when discussing weird poems,
compared to other types of poetry because __________ .

vi Based on the reading of the passage, examine, in about 40 words, how studying poetry can be like exploring
a new city.

vii What is the message conveyed by Hina's experience, in the following case?
Hina spends hours trying to analyze a poem for her assignment and finally feels a sense of accomplishment
and pride, once she understands.
A. Only those with natural talent for poetry should engage with it.
B. Persistence makes studying poetry a rewarding pursuit.
C. Study of poetry is guaranteed to impress others.
D. The efforts of studying poetry is inversely proportional to the rewards gained.

viii State whether the following lines display an example of a simple / complex / emotionally charged /
downright weird, poem.

The sun rises in the east,


A new day begins, a fresh start.
Birds chirp, nature wakes up,
A peaceful feeling in my heart.

Passage 2
Read the following text. 10 m
1. Reduction in green areas has caused various environmental problems. People squeezed between concrete
structures are looking for various ways to meet their longing for green. One of the ways to do so, is vertical
gardens and green walls. Vertical gardening is a unique method of gardening where plants are grown in a
vertical position or upward, rather than in the traditional method of planting them on the ground.

2. The purpose of vertical gardens and green walls, which arises from the studies of different disciplines
(landscape architects, architects, engineers, etc.), is to close the cold image of concrete and increase the
visual value. In these systems, nature and structures are integrated, and thus, urban areas and the desired
environment have become intertwined.

3. Vertical garden case studies often show that , though functionality should be in the foreground, when
vertical gardens are planned, they are generally made as aesthetic elements in the city's underpasses and
city squares, and decorative elements in residences, without seeking functionality.

4. Experts support that the visual quality and evaluation of landscape architecture is determined based on
the satisfaction of the users. Hence, a survey questionnaire was prepared for residents of varied age groups
from of a metropolitan city . The given Table 1, displays these responses:

3
5. The study acknowledged that vertical gardening has the potential to transform urban spaces into green,
sustainable areas, and further research should explore the impact of vertical gardening on the environment
and human well-being.

Answer the following questions, based on the passage above:


i. Complete the following analogy appropriately, based on your understanding of paragraphs 1 & 2.
We can say that the situation of people living in concrete structures is comparable with a fish living in a
fishbowl, and the need for vertical gardens to the need for decorations in the fishbowl because
_______________.

ii. Fill the blanks with the appropriate option from those given in brackets, based on your understanding of
paragraph 2.
The statement that , urban spaces have become more closely connected with the desired natural
surroundings through the incorporation of nature and structures in vertical gardens and green walls, is a
___________ (fact/ opinion) because it is a _____________ (subjective judgement/ objective detail).

iii. Justify the following, in about 40 words.


While the survey results suggest that vertical gardens may be effective in improving the quality of life in
urban areas, further research and evaluation may be necessary to fully understand their effectiveness and
potential drawbacks.

iv. Based on the survey results, which two concerns should a city government, looking to install vertical
gardens, address?

v. In Table 1, the statement 3, "Vertical gardens increase air quality - indoors and outdoors," received the
most neutral responses from participants, with 51 respondents indicating a neutral stance. State any one
inference that can be drawn from this.

vi. Select the option that correctly displays what ‘intertwined’ signifies. (Reference-Paragraph 2)

A. (i), (iv) and (v)


B. Only (ii)
C. Only (iii)
D. (ii) and (v)

vii. Infer one benefit and one drawback of vertical gardening, in comparison to other solutions, such as
community gardens or parks. (Answer in about 40 words)

viii. Which of the following is the main takeaway from the study mentioned in the passage?
A. Vertical gardening has minimal impact on the environment or human well-being.
B. Vertical gardening is a sustainable practice that can transform urban spaces into green areas.
C. The impact of vertical gardening on the environment and human well-being has already been thoroughly
explored.
D. The study needs to include experts from horticultural firms to offer any recommendations for further
research.

Passage 3

Read the following text. 10 m


1. Starting Monday, the country's low-cost Mars mission with the red planet for an extended period will
enter the "blackout" phase snapping communication with the satellite. From June 8 to 22 the Sun will block
Mars from the Earth snapping communication with the satellite.

2. A senior Indian Space Research Organisation official said. "This will be for the first time that there will
be a communication break for such a long period of about 15 days. During this period, there will be no
communication with the satellite", he added.

5
3. Expressing confidence about regaining control over the satellite once the blackout phase is over, he said,
"the scenario has been tested and the line of communication will be established." The spacecraft's life has
been extended for another six months in March due to surplus fuel.

4. Stating that the spacecraft has been 'configured' for the blackout, the ISRO official said, "we are not
sending any commands to the spacecraft now, till 8th June few hours of signals will be sent by the
spacecraft that will be for about two to three hours per day". In May next year, the mission will have to go
through a similar phase once again, if there is another extension of mission life when the Earth will come
between the Sun and Mars.

5. Scripting space history, India on 24th September last successfully placed its low-cost Mars spacecraft in
orbit around the red planet in its very first attempt, breaking into an elite club of three countries.

Answer the following questions, based on the passage above:


i Infer one reason for the following, based on information in paragraph 1.
The ‘blackout’ phase is significant……………

ii Choose the appropriate option to fill in the blank.


From paragraph 1&2, we can infer that there will be………
1. no communication with the people
2. blackout phase
3. satellite will get no communication
4. moon will block the earth
5. the communication break will be for 15 days.

(A) 1, 2 & 3 (B) 1, 3 & 4


(C) 2, 3 & 5 (D) 3, 4 & 5

iii According to ISRO official till 8th June, when Sun will block Mars _________
signal/signals per day will be sent to the spacecraft.
(A) no (B) considerable
(C) indefinite (D) only few

iv Complete the following sentence.


The ISRO officials are ___________about their control over the spacecraft after the blackout phase.

v The spacecraft’s life was extended by six months because of

(A) extra fuel (B) incomplete work


(C) lack of communication amongst ISRO officials
(D) technical problems in its landing

vi Read the following sentence:


(a) The blackout was a sudden development
(b) Because of this, the officials are very nervous about the success of the spacecraft.

(A) Both A & B are true (B) Both A & B are false
(C) A is true and B is false (D) A is false and B is true

vii Complete the sentence.


The word ‘fascination’ in the opening sentence means the same as ______.
viii Throw light on India’s script in space history in about 40 words.
Passage 4
Read the following text. 10 m
1. Tourists to Jammu and Kashmir have another attraction – a floating post office on the Dal Lake in
Srinagar, the first in the country. ‘Floating Post Office, Dal Lake’ – claimed to be the only one such post
office in the world – is built on an intricately carved maroon houseboat, fastened on the western edge of the
Dal Lake.

2. This post office lets you avail of all the regular postal services available in the country while being
afloat. The seal used on everything posted from Floating Post Office is unique – along with the date and
address, it bears the design of a boatman rowing a shikara on the Dal Lake. The special feature of this post
office is that letters posted from here carry a special design which has the picturesque
scenery of Dal Lake and Srinagar city. These pictures reach wherever these letters are posted to and hence
promote Kashmir as a tourist destination across the world.

3. This is actually a heritage post office that has existed since British times. It was called Nehru Park Post
Office before 2011. But then the chief postmaster John Samuel renamed it as ‘Floating Post Office’.

(4) The post office’s houseboat has two small rooms – one serves as the office and the other a small
museum that traces the philatelic history of the state postal department. It has a shop that sells postage
stamps and other products.

(5) But for the locals, Floating Post office is more than an object of fascination. 1-2 crore is deposited per
month in Floating Post Office by communities living in and around the Dal Lake. The lake has several
islets that are home to more than 50,000 people.

(6) The greatest fear is the recurrence of 2014 like floods in which the houseboat had gone for a toss
uncontrollably pushed by the flood. Rescue teams had to anchor it using special mechanism in a nearby
highland. Then it was brought back on the Dal after the water receded. The biggest boon is that at no time
of the year do you need a fan in this post-office!

Answer the following questions, based on the passage above:


i Complete the given statement.

7
Before 2011, this post office was called __________
ii What is unique about the seal used in the floating Post Office? Justify your response in about 40 words.

iii How is the post office described by the writer?

iv Why is the Floating post Office more than a fascination for the locals?

v ___________ is claimed to be the only one such in the world.


(A) the museum of the state postal department
(B) the shop that sells postage stamps
(C) Floating Post Office situated on Dal Lake
(D) All of the above

vi Complete the given statement.


________________renamed the post office as ‘floating post office’.

vii The special feature of this post office is that letters posted from here__________
(A) have the design of a houseboat
(B) have the design of snow-capped mountains
(C) have the design of Dal Lake (D) have the design of local handicrafts

viii What function does the picturesque scenery of Dal Lake and Srinagar city fulfil? Answer in about 40
words.

Passage 5
Read the following text.
1. Hiking is a great source of pleasure to us besides being beneficial for health. Once we leave the crowded
streets of a city and go out for a walking tour away from the mad world, we really feel free. The open air
has a bracing effect on the mind. The congestion of the city, the uproar and tumult, the intolerable noise of
traffic, the hectic daily routine, all these are forgotten and the mind is at ease.

We then feel like running, leaping, singing and laughing. We travel merrily mile after mile in the company
of friends and associates.

2. Hiking takes us in the midst of nature. The sight of waterfalls, flowers, streams, trees and bushes is
pleasing. A connect is established between us and nature. (Various sounds of nature, like the murmur of a
brook or the song of a bird, acquire a new meaning and significance to us.)

3. You enjoy the beauty of nature. Minute observations like a snake casting its slough, a mouse peeping out
of its hole, a squirrel leaping about on the branches of a tree, a bird flying past us, all these are noticed and
they arouse our interest. We have no business worries, no fear of the examination, no anxiety about the
home. We have leisure to stand, to walk and talk. It is more thrilling and pleasurable to hike in a
mountainous region than in the plains. The excitement of climbing up to the top of a hill, the adventure of
coming down a slope, the grandeur of sunset behind a mountain – All these sights lend a rare charm and
interest to our journey.

4. We walk along a zigzag motor road or cut across a mountain path in search of adventure. We may have
bright sunny weather or might get caught in a shower of rain. We may look below us into the yawning
chasm or up at the mountain peak. The feeling of unlimited freedom makes our hearts leap with joy.
5. Hiking is one of the healthiest sports. It ensures a complete escape from the urgent and busy activities of
life and therefore gives solace to our brain. It regains its lost energy and is able to do twice as much work as
before.
6. The fresh air, beautiful mountains, majestic trees, chirping sound of birds make one's mind and soul at
peace with nature.

Based on your understanding of the passage, answer the questions below:


(i) To go out for a walking tour is
(a) to stroll (b) to ramble
(c) to hike (d) to saunter

(ii) Why does the writer say that the mind is at ease when you hike?
(a) because hiking is an inexpensive activity.
(b) because it brings families together.
(c) because of intolerable noise of traffic.
(d) because the depressing daily routine is forgotten.

(iii) Complete the following with a phrase:


The various sounds of nature acquire ..............

(iv) Infer one reason for the following based on information in paragraph 2.
A contact is established between us and nature

(v) Complete the following analogy correctly with a word from paragraph 2.
aroma : cooking : fragrance : .............

(vi) Give one reason why it is a pleasure to hike in the mountains than in the plains.
(a) because observation is sharpened.
(b) because of the excitement of climbing up and
adventure of coming down.
(c) because it is leisure to stand, to walk
(d) because there are no worries.

(vii) Hiking gives the brain, the rest it needs because


(a) it is a short time activity.
(b) it is one of the healthiest sports.
(c) it makes us sleep peacefully.
(d) it is an escape from our busy schedule.

(viii) List any two examples of minute observations you make while on a hike.

(ix) Supply one point to justify the following:


Hiking gives us a feeling of unlimited joy.

(x) Substitute the word 'leap with joy' with one word similar in meaning in the following sentence from the
passage.
The feeling of unlimited freedom makes our hearts leap with joy.

Passage 6
Read the following text.
9
1. Necessity is indeed the mother of invention. When areas in and around Leh began to experience water
shortages, life didn't grind to a halt. Why? Because a retired civil engineer in the Jammu and Kashmir
Government came up with the idea of artificial glaciers.
2. Ladakh, a cold desert at an altitude of 3,000-3,500 meters above sea level, has a low average annual
rainfall rate of 50 mm. Glaciers have always been the only source of water. Agriculture is completely
dependent on glacier melt, unlike the rest of river/monsoon-fed India. But over the years, with increasing
effects of climate changing, rainfall and snowfall patterns have been changing, resulting in severe shortage
an drought situations. Given the extreme winter conditions, the window for farming is usually limited to
one harvest season.

3. It is located between the natural glacier above and the village below. The one closer to the village and
lowest in altitude melts first, providing water during April-May, the crucial sowing season. Further, layers
of ice above melt with the increasing temperature, thus ensuring continuous supply to the fields. Therefore,
farmers have been able to manage two crops instead of one. It costs about 1,15,000 and above to create a
glacier.

4. Fondly called the glacier man, he has designed over 15 artificial glaciers in and around Leh since 1987.
In recognition of his pioneering effort, he was conferred the Padma Shri by the President of India.

5. There are a few basic steps followed while creating an artificial glacier. River or stream water at high
altitude is diverted to a shaded area of the hill, facing north, where the winter sun is blocked by a ridge or a
mountain range. At the start of winter i.e., in November, the diverted water is made to flow on sloping hills
facing distribution channels. Stone embankments are built at regular intervals which impede the flow of
water, making shallow pools which freeze, forming a cascade of ice along the slope. Ice formation
continues for 3-4 months resulting in a large accumulation of ice which is referred to as an 'artificial
glacier'.

Based on your understanding of the passage, answer the questions below:


(i) Infer one reason for the following, based on information in paragraph 1.
Areas in and around Leh began to experience water shortage but life didn't grind to a halt.

(ii) Select from the passage the appropriate option to fill in the blanks:
Agriculture is completely dependent on ............ unlike the rest of river/monsoon-fed India.
(a) rainfall pattern
(b) climate change
(c) glaciers melt
(d) extreme winter conditions

(iii) Complete the following analogy correctly with a word/phrase from


paragraph 3:
icing : cake :: layers : ..............
(iv) Select the correct option to complete the following sentence:
At the start of winter the diverted water is made to flow .............
(a) on sloping hills facing distribution channels.
(b) on high altitude
(c) one ice-cold water level.
(d) on mountain range.

(v) From the chart select the months of water surplus.


(a) January, February.
(b) November, December.
(c) July, August, September.
(d) March, April.

(vi) Fill in the blank by selecting from the passage the correct option:
The ............... have been the only source of water in areas in and around Ladakh.
(a) glaciers (b) rivers
(c) streams (d) water springs

(vii) The word 'located' in paragraph 3 means:


(a) found (b) locally found
(c) situated (d) adapted

(viii) Write any two steps followed while creating the glaciers.

(ix) List one reason why artificial glaciers seem the best option.

(x) Select the most suitable title from the following for the passage:
(a) Water Shortage (b) New Ways to Pool Water
(c) Artificial Glacier (d) River Beds
SOLUTIONS

Discursive Passage 1
NOTE: The information in parenthesis is extra information given to help you to solve the passage.
i. C. Recognizes the challenges of studying poetry but also acknowledges the rewards it offers.
[Throughout the initial paragraph, the author acknowledges the difficulties of studying poetry, but also
notes the emotional resonance and sense of accomplishment that can be derived from analysing complex
works. Therefore, option C is the correct answer.]
ii. The tone - critical
Rationale - The writer seems to be expressing their scepticism and dissatisfaction with poems that are
overly obscure and difficult to understand. The writer suggests that such poems may not be worth the effort
and compares them unfavourably to solving a riddle. [Overall, the tone suggests a frustration with poetry
that prioritizes obscurity over clarity and meaning.]
OR
The tone - sarcastic
The statement "if solving a riddle is what was intended, then playing Sudoku is a better option" suggests
that the writer is not impressed with the level of complexity in some poetry. Additionally, the phrase "led to
ponder if obscurity was the goal" implies that the writer believes some poets may intentionally make their
work difficult to understand, which can be frustrating for readers.[ The sarcastic tone in these lines is likely

11
a reflection of the writer's opinion that poetry should not be needlessly complex and should strive to
connect with readers.]
iii. ... it creates a powerful emotional impact and enhances the reader's understanding of the intensity of
emotion that can be conveyed through poetry.
iv. (a) intriguing and off-putting
(d) simple and challenging
[They represent opposite concepts, with "simple" meaning easy, uncomplicated, or straightforward, while
"challenging" means difficult, demanding, or requiring effort. In the given passage, the author uses both
terms to describe different types of poetry that a reader may encounter.]
v. ...the author acknowledges that weird poems can be off-putting, but also appreciates their uniqueness and
the challenge they present to readers [hence, the author is being neutral and objective with no bias]
vi. Just as exploring a new city requires an open mind and a willingness to embrace the unexpected,
studying poetry requires an openness to different styles and approaches, and a willingness to be challenged
and surprised by what you find.
vii. B. Persistence makes studying poetry a rewarding pursuit.
[Q. is rooted in paragraph (4). Hina's experience of finally unlocking a poem's meaning and feeling a sense
of accomplishment shows that poetry can be rewarding if one is persistent.]
viii. Simple
[This verse is simple in terms of its language, structure, and content. It describes a natural scene, which is
easy to comprehend and has a peaceful effect on the reader.]

Case-based factual passage 2


i. ... a fish needs decoration such as plants or rocks, to simulate a natural environment and improve its
quality of life, people living in urban areas need green spaces such as vertical gardens to improve their
physical and mental wellbeing.
[ The analogy highlights the importance of providing a natural and healthy environment for living beings,
regardless of their habitat, and emphasizes the need to explore creative solutions to meet this need.]
ii. opinion; subjective judgement
[This statement expresses a viewpoint or belief about the benefits of incorporating nature and structures
through vertical gardens and green walls in urban spaces. While there may be evidence to support this
opinion, such as research on the positive effects of greenery on well-being or the visual appeal of vertical
gardens, it is still ultimately a subjective judgment rather than an objective fact that can be proven or
disproven.]
iii. True, because the majority of participants strongly agreed or agreed that vertical gardens can improve
the quality of life, increase air quality, and have a relaxing and calming effect. However, the survey does
not provide information on the long-term effectiveness of vertical gardens or their potential impact on other
aspects of urban life, such as traffic or community engagement.
iv. A city government looking to install vertical gardens should consider addressing these concerns:
cost of installation and maintenance [should be reasonable]
areas of installation should be where driver distraction is less of an issue.
[Both need to be mentioned as the survey reflects these]
v. Possible inferences: (any one)
Participants may not be fully informed or aware of the effects of vertical gardens on air quality, which
could contribute to the neutral response. [This suggests a need for more education and awareness
campaigns around the benefits of vertical gardens.]
The perceived effect of vertical gardens on air quality may vary based on the specific location or
environment in which they are installed. [This could explain the higher number of neutral responses, as
participants may not have enough information to judge the statement in a generalized manner.]
It is possible that participants did not have a strong opinion or preference regarding the effect of vertical
gardens on air quality, which resulted in a neutral response. [This could indicate a need for more targeted
research to understand the potential benefits and limitations of vertical gardens in improving air quality.]
vi. B. Only (ii)
[Cambridge dictionary - to twist or be twisted together, or to be connected so as to be difficult to separate],
Hence, Option B.
vii. Advantages of vertical gardens over other green spaces-
They can be installed in small spaces, making them ideal for urban areas with limited space.
They also offer aesthetic and design benefits, as they can be customized to fit specific architectural styles
or preferences.
Disadvantages of vertical gardens over other green spaces-
May not provide as much space for outdoor recreation as community gardens or parks do.
The cost of installation and maintenance may be higher than for other types of green spaces.
viii. B. Vertical gardening is a sustainable practice that can transform urban spaces into green areas.
[The passage clearly states that the study acknowledged the potential of vertical gardening to transform
urban spaces into green, sustainable areas. This suggests that vertical gardening is seen as a sustainable
practice that can have a positive impact on the environment and human well-being. The other answer
choices are incorrect as they either contradict the information given in the passage (A and C) or are not
relevant to the passage (D).]

Discursive Passage 3
i. The blackout phase is significant because it will lead to snapping all communication with satellite that is
on a mission to Mars.
ii. option (C) is correct
iii. option (D) is correct
iv. confident
v. option (A) is correct
vi. option (B) is correct
vii. enchantment
viii. On September 24, India successfully placed its low-cost mars spacecraft in orbit around the red planet
in its very first attempt. This broke an elite-club of three countries, thus scripting India’s space history.

Passage 4
i. Nehru Park Post Office
ii. The seal is unique because it bears the design of a boatman rowing a Shikara unlike in any other post
office in the world.
iii. The post office is described as an intricately carved maroon houseboat moored on the western edge of
Dal Lake.
iv. The locals deposit crores of money in the post office, so it is just not only an object of fascination for
them
v. option (C) is correct
vi. John Samuel
vii. option (C) is correct
viii. The pictures reach wherever the letters are posted to, and hence promote Kashmir as a tourist
destination across the world.

Passage 5
(i) Option (c) is correct
(ii) Option (d) is correct
(iii) Acquire a new meaning and significance to us.
(iv) because hiking takes us in the midst of nature and the sight of waterfalls, flowers, streams, trees and
bushes is pleasing.
(v) flowers
(vi) Option (b) is correct
13
(vii) Option (d) is correct
(viii)Two examples of minute observations made while on a hike are:1. A snake casting its slough.
2. A mouse peeping out of its hole.
(ix) Hiking gives a feeling of unlimited joy as it gives us unlimited freedom and the sights of nature lend a
charm to our journey.
(x) exult

Passage 6
Ans. (i) because a retired civil engineer in Jammu and Kashmir Government came up with the idea of
artificial glaciers.
(ii) Option (c) is correct
(iii) ice
(iv) Option (a) is correct
(v) Option (c) is correct
(vi) Option (c) is correct
(vii) Option (a) is correct
(viii)(1) River or stream water at high altitude is diverted to a shaded area of the hill, facing north, where
the winter sun is blocked by a ridge or a mountain range.
(2) At the start of winter i.e., in November, the diverted water is made to flow on sloping hills facing
distribution channels.
(ix) One reason why artificial glaciers seem like the best option is that they provide a continuous supply of
water for agriculture even in areas where the natural glacier has melted.
(x) Option (c) is correct

Source www.shaalaa.com

GRAMMAR AND WRITING SECTION

Subject verb agreement

The term ‘subject-verb agreement’, just like the name suggests, refers to the agreement between the subject
and the verb. This is mainly with reference to singular and plural.

Rule 1
The first rule is what we have already discussed – the use of a singular verb with a singular subject and a
plural verb with a plural subject. The subject can be a noun, a pronoun or even a noun phrase. If it is a
pronoun, the subject-verb agreement is done with reference to the person of the pronoun.

For example:
1. Rachel spends her free time listening to music. (Singular subject with singular verb)
2. Blaine and Kurt play the piano. (Plural subject with plural verb)
3. She likes to have a dessert after every meal. (Third person singular pronoun with singular subject)

Rule 2
The use of ‘have’ and ‘has’ in the present perfect tense, the present perfect continuous tense and as a main
verb is also dependent on the subject. All singular subjects use ‘has’ and all plural subjects use ‘have’.

For example:
1. I have a younger brother.
2. You have taken the wrong cut.
3. Swetha has a pet dog.
4. William Shakespeare has written around 37 plays.
Rule 3
Compound subjects combined using the conjunction ‘and’ take a plural verb.

For example:
1. Krish and Radha are on their way to the airport.
2. Caren, Sheela and Akash have completed their assessments.

Rule 4
When more than one noun is joined by the conjunction ‘or’, the subject is considered to be singular and a
singular verb is used.

For example:
1. Celery or spring onion works fine.
2. Your mom or dad has to be here in an hour.

Rule 5
Sentences with pronouns such as anybody, anyone, no one, somebody, someone, everybody, everyone,
nothing and nobody are treated as singular subjects and will therefore use a singular verb.

For example:
1. Nobody has understood anything.
2. Everyone was happy with the outcome.
3. Nothing fits me well.

Rule 6
For sentences using ‘either..or’ and ‘neither..nor’, the verb should agree with the noun or pronoun that
comes just before it.

For example:
1. Neither Ricky nor Gina is here yet.
2. Either the teacher or the students have to take an initiative to keep the classroom clean.

Rule 7
When sentences have subjects like police, news, scissors, mathematics, etc. (nouns that are plural by
default), the verb used should be plural.

For example:
1. The news of demonetisation shocks the entire nation.
2. The police have been looking for the culprits.

Exercise:
Choose the correct form of the verbs given in the brackets in the following sentences:
1. Each of these producers……………………… his own advantage.(has/have)

2. Ten miles……………………… long distance. (is/ are)

3. Many students ………………….. hard to pass his entrance exam. (try/ tries)

4. The furniture in his house………………….. impressive.(look/looks)


15
5. Few students……………….. present in the class today.(is/ are)

6. Ritesh, my best
friend…………………… leaving for Japan next week.(is/ are)

Answers.
1. has
2. is
3. tries
4. looks
5. are
6. is

Identify the error and supply the correction in the given format.

7. Most of my friends is government employees.

Error Correction

8. The level of intoxication vary from subject to subject.

Error Correction

9. Either boys or girla has telling lies.

Error Correction

10. A number of peopl was reported to be missing in the train accident at Jhansi.

Error Correction

Answers:
1. is —-- are
2. vary—--varies
3. has—--- are
4. was—---were

Determiners
What is a determiner?

A determiner is a word that appears before a noun and specifies something about the number, definiteness,
or ownership of the noun.

Types of determiners

Practice Exercises

Fill in the blanks with suitable determiners .

1. She was ___________ patient.

(enough/ very/lot)

2. There is___________ food at the celebration for everyone.

(enough/ very/lot)

3. He scored a ___________ of runs during the game.

(enough/ very/lot)

4. I bought___________ vegetables from the market.

(some/ very/alot)

5. ___________my friends are going to a movie tonight.

(Few/ All /A Alot)

6. I’m feeling a ___________bit sleepy.

(enough/ little/very)

7. They stayed a few___________days in Bangalore.

17
(a little/ very/few)

Answers

1. very
2. enough
3. lot
4. some
5. All
6. little
7. few

Choose the correct determiner for the sentence.

1.I think the ___________ man in the lineup is the thief.

a. third
b. mine
c. this

2. ___________ the team didn’t show up to practice.

a. My
b. This
c. Half

3.___________ Lina nor John has met the new neighbors.

a. My
b. Neither
c. Each

4.___________ height is the Empire State Building?

a. What
b. When
c. Which

5. There are already ___________ people waiting for a table at this restaurant.

a. which
b. third
c. several

6. The car was parked in front of ___________ dilapidated brick building.

a. the
b. an
c. a

Answers:
1. This
2. Her
3. The
4. My
5. Whose
6. a

Modals

What are Modals?


Modal verbs show possibility, intent, ability, or necessity.

The modal verbs are:

Exercise.

1.I ___ run fast to catch the train.

(should/would/will)

2.I ___ bring milk on my way home.

(must/can/will)

3.He ___ faces a lot of problems getting this approved.

(should/could/might)

4.She ___ be at school on time.

(must/can/will)

5.He ___ does what makes him happy.

(must/could/might)

19
6.___ I get you a glass of water?

(Must/Can/May)

7.He ___ takes part in the occasion.

(must/can/will)

8.We ___take care of our surroundings.

(must/can/will)

Answers

1. Should
2. Can
3. Might
4. Should
5. Must
6. Can
7. Will
8. Must

Identify the error and supply the correction in the given format.

9. One could repay all their debts.

Error Correction

10. Shall you have hot chocolate?

Error Correction

11. The child should be taken to hospital immediately

Error Correction

12. We may take care of our parents

Error Correction

Answers
9 . could —must
10.shall—will.
11.should—.must
12.may—---.ought to

Tenses
A tense is a form of the verb that allows you to express time. The tense of the verb tells us when an event or
something existed or when a person did something. Past, present, and future are the three main types of
tenses.

1.The following table shows the different forms of the verb:

2. Forms of the present Simple Tense:


Thus we have seen that the Simple Present Tense is formed by using the plain infinitive. But -s or -es are
added to the bare infinitive (i.e. infinitive without ‘to’) for the third person singular (He, She) and singular
noun (Nitu)
● We form the negative sentences by using doesn’t or don’t before the main verb.
● The interrogative sentences are formed by using do or does before the subject.
● The negative interrogative sentences are formed by using do or does before the subject and not after
the subject.

But the short forms ‘don’t’ and ‘doesn’t’ come before the Subject

Present Simple
We use Present Simple time to talk about things in general. We are not thinking only about now. We use it
to say that something happens all the time or repeatedly or that something is true in general. Here it is not
important whether the action is happening at the time of speaking

Examples:
● I take my breakfast every day.
● We go to school.

Present Continuous
21
We use Present Continuous tense to talk about something that is happening at or around the time of
speaking. The action is not finished. In such sentences is/am/are + verb + ing is used to show continuation.
Examples:
● Girls are playing.
● I am watching TV.

Present Perfect
Present Perfect tense is used to give information about an action that has recently been completed. It is also
used to talk about some action in the past that has a result now. In such sentences verb ends with have/has +
verb + ed/t/en/ne forms.
Examples:
● Tom has lost his key.
● I have forgotten your name.

Present Perfect Continuous


Present Perfect Continuous tense is used to indicate an activity that continues from the past until now
repeatedly or an activity that has recently stopped. In these sentences have/ has + been + v + ing form is
used.
Examples:
● John looks sunburnt. He has been working under the sun since morning.
● Your clothes are very dirty. What have you been doing?

Past Tense
Simple Past
● I/You/He/She/It/We/They played. – (Affirmative)
● I/You/He/She/It/We/They didn’t play. – (Negative)
● Did I/You/She/It/We/They play? – (Interrogative)
All these sentences are in simple past.
We use Past Simple for:
● Action completed in the past
● Actions which happened at a specific point in time.
● Past habits and past actions that happened immediately after the other.
Now look at the following sentences:
● I/He/She/It was playing. – (Affirmative)
● We/You/They were playing. – (Affirmative)
● I/He/She/It wasn’t playing. – (Negative)
● We/You/They weren’t playing. – (Negative)
● Was/I/He/She/It playing? – (Interrogative)
● Where were/We/You/They playing? – (Interrogative)

All these sentences are in Past Continuous.


We use past continuous for:
● An action that was in progress at a stated time in the past.
● A past action which was in progress when another action interrupted it.
● Two or more actions were happening at the same time.

Let’s read the following sentences:


● Rohit had already left when we arrived at the party.
● He had broken his leg and it was still hurting.
● Everything had seemed normal at first.

All these sentences have been written in Past Perfect Tense.


We use it to talk about an action which finished before another action in the past.
We use certain time expressions with past perfect: before, already, after, just, when, never, etc.
Now read the following sentences:

I had been watching TV for an hour when I remembered I had forgotten to call my friend.
She had been painting her room and her clothes were covered in paint.

These sentences have been written in Past Perfect Continuous Tense.


We use it to talk:
● for a past action which started and finished in the past before another past action, putting emphasis
on the duration.
● for an action which lasted for some time in the past and the result was still visible in the past..
Time expressions used: for, since, until, etc.

Future Tense
● Let us read the following sentences:
● We will go to the zoo tomorrow.
● He will be on leave next week.
All these sentences indicate that we use will to talk about an event in progress at some point in future.

The 4 Future
Examples Uses
Tenses
simple future ● I will go. The simple future tense is used for an action that
tense ● We will celebrate our will occur in the future.
anniversary by flying to New
York.
future ● I will be going. The future progressive tense is used for an
progressive ● The Moscow State Circus will be ongoing action that will occur in the future.
tense performing in Cheltenham for
the next 3 weeks.
future perfect ● I will have gone. The future perfect tense is used to describe an
tense ● By the time you arrive, we will action that will have been completed at some
have finished the meal and the point in the future.
speeches.
future perfect ● I will have been going. The future perfect progressive tense is used for
progressive ● In July next year, you will have an ongoing action that will be completed at
been studying for three years. some specified time in the future.

However, there are different modes in English that can be used to refer to incidents that occur at some time
in future. This mode of reference is called as future time reference. Such sentences have constructions
based on different structures.

They are as follows:


Simple Present Tense
● The Commonwealth Games begin from 2nd October.
● If he works hard like this, he can pass.

Present Continuous Tense: It is used to refer to future events that have been planned before.
23
● We are expecting the queen to inaugurate the games this month.
● I’m sorry I can’t attend the wedding. I’m meeting the director this evening.
Use of Going to
● We are going to shift to Shimla very soon.
● Suchitra is going to marry Harish.

Be + about to + infinitive
● The class is about to start.

Use of Will/Shall
It is used to make a prediction about future, or in advertisements, etc.
● India will win at least one gold in boxing.
● Their souls shall rest in peace.

Be + To + V
● The teacher is to deliver a talk on study skills.
Exercises
Fill in the blanks with the correct form of the verbs by choosing right option.
1. The police………………….. four thieves last night.
a) catches b) caught c) has caught d) catching
2. I ………………………. food when he came in.
a) cooked b) had cooked c) was cooking d)am cooking
3. It………………… since 9 o’clock.
a) has been raining b)rained c)is raining d)was raining
4. I certainly…………………… my colleague if I had been there.
a) had helped b) have been helped c)would help d) would have helped
5. He is a very rude person, I ………………help him.
a) won’t b)didn’t c) couldn’t d) haven’t

Select the option that identifies the error and supplies the correction for the given line from a news report.
a. MV Ganga Vilas cruise don’t get stuck and the reports claiming so are absolutely false.

Option No. Error Correction


(i) don’t didn’t
(ii) get got
(iii) claiming claim
(iv) are is
b.I am going to school with my brother on foot every day.

Opion No. Error Correction


(i) on by
(ii) with for
(iii) am going go
(iv) going gone

Fill in the blanks by using the correct form of the verb in the bracket.
1. A few months ago a young man from Punjab ……………….(decide) to go to Canada.
2. A forty year old man ……………………(kill) and six others injured when the van overturned after
colliding with atruck last evening.
3. My father …………. (leave) before I reached home from the school.
Identify the error and supply the correction in the given format.
a) People of diverse cultures lives in India
Error Correction

b) My mother was cooking food when my friends had come to call me for cricket match.
Error Correction

c) My brother watched TV while I was doing my homework.


Error Correction

d) If I were a king, I will change the entire dynasty.


Error Correction

e) I scored at least one goal in the upcoming final match.


Error Correction

Answers:

Fill in the blanks with the correct form of the verbs by choosing right option.
1. b) caught
2. c) was cooking
3. a) has been raining
4. d) would have helped
5. a) won’t
Select the option that identifies the error and supplies the correction for the given line from a news report.
1. (i) don’t didn’t
2. (iii) am going go
Fill in the blanks by using the correct form of the
verb in the bracket.
1. decided
2. was killed
3. had left
Identify the error and supply the correction in the given format.
a)
Error Correction
lives live
b)
Error Correction
had come came
c)
Error Correction
watched was watching
d)
Error Correction

25
will would
e)
Error Correction
scored will score

Direct and Indirect Speech

1. Direct and Indirect Speech:


The words spoken by a person can be reported in two ways—Direct and Indirect.

When we quote the exact words spoken by a person, we call it Direct Speech.

● Sohan said to Mohan, “I am going to school.”


The exact words spoken by Sohan are put within inverted commas. But when we give the substance of
what Sohan said, it is called the Indirect Speech or reported speech.

2. Rules for Changing Direct Speech into Indirect Speech:


● In the Indirect speech, no inverted commas are used.
● The conjunctions that, if, whether, are generally used after the reporting verb.
● The tense of the reporting verb is never changed.
● The reporting verb changes according to sense: it may be told, asked, inquired,etc.

3. Rules for the Change of Pronouns:


● The first person pronouns (I, me, my, we, us, our) in the reported speech change according to the subject of
the reporting verb.
● The pronouns of the second person (you, your, yourself) in the reported speech change according to the
object of the reporting verb.
● The pronouns of the third person do not change.
For example:
1. He said, “I like the book.”
He said that he liked the book.
2. He said to me, “Do you like the book?”
He asked me if I liked the book.
3. He said, “He likes the book.”
He said that he liked the book.
4. Changes in words expressing nearness, time, auxiliaries, etc.

That

th
at
S
p
e
ec
h
2
(S
ta
te
m
5. Change in Tenses: e
nt
● If the reporting verb is ins,the present or the future tense, the tense of the reported speech is not changed:
1. Satish says, “I am flying
Q a kite.”
2. Satish says that he isuflying a kite.
3. Satish will say, “I want a glass of milk.”
es
4. Satish will say that he wants a glass of milk.
ti
● If the reporting verb is inothe past tense, then the tense of the reported speech will change as follows:
ns
,
R
e
q
u
es
ts
a
n
d
O
rd a historical fact, a universal truth or a habitual fact. tense of the direct speech
● If the direct speech expresses
will not change: er
Direct : He said, “Honesty s) is the best policy.”
Indirect : He said that honesty
C is the best policy.
Direct : He said, “Thehsun rises in the east.”
Indirect : He said that thea sun rises in the east.
n
6 . Changing Statements into Indirect Speech:
ge
27
th
e
di
● The reporting verb “said to’ is changed to ‘told, ‘replied’, ‘‘remarked’,
● The reporting verb is not followed by an object, it is not changed.
● The inverted commas are removed. The conjunction that is used to connect the reporting clause with the
reported speech.
● The rules for the change of pronouns, tenses, etc. are followed.
Direct : Ramu said, “I saw a lion in the forest.”
Indirect : Ramu said that he had seen a lion in the forest.
Direct : Satish said to me, “I am very happy here.”
Indirect : Satish told me that he was very happy there.
Direct : He said, “I can do this work.”
Indirect : He said that he could do that work.

7.Rules for the Change of Interrogative (Questions) sentences:


● The reporting verb ‘say’ is changed into ask, inquire,
● The interrogative sentence is changed into a statement by placing the subject before the verb and the full
stop is put at the end of the sentence.
● If the interrogative sentence has a wh-word (who, when, where, how, why, etc) the wh- word is repeated in
the sentence. It serves as a conjunction.
● If the interrogative sentence is a yes-no answer type sentence (with auxiliary verbs aw, are, was, were, do,
did, have, shall, etc), then if or ‘ whether’ is used as a conjunction.
● The auxiliaries do, does, did in a positive question in the reported speech are dropped.
● The conjunction that is not used after the reporting clause.

Direct : I said to him, “Where are you going?”


Indirect : I asked him where he was going.
Direct : He said to me, “Will you go there?”
Indirect : He asked me if I would go there.
Direct : My friend said to Deepak, “Have you ever been to Agra?”
Indirect : My friend asked Deepak if he had ever been to Agra.
Direct : I said to him, “When will you go there?”
Indirect : I asked him when he would go there.
Direct : He said to me, “How is your father?”
Indirect : He asked me how my father was.

8. Changing Commands and Requests into Indirect Speech:


● In imperative sentences having commands, the reporting verb is changed into command, order, tell, allow,
request,etc.
● The imperative mood is changed into the infinitive mood by putting to, before the verb. In case of negative
sentences, the auxiliary ‘do’ is dropped and ‘to’ is placed after ‘not:
Direct : The captain said to the soldiers, “Attack the enemy.”
Indirect : The captain commanded the soldiers to attack the enemy.
Direct : I said to him, “Please bring me a glass of water.”
Indirect : I requested him to bring me a glass of water.
Direct : I said to my friend, “Please lend me your book.”
Indirect : I requested my friend to lend me his book.
9. Exclamatory Sentences (Strong Feelings)
Change the reporting verb ‘said’ or ‘said to’ into “exclaimed with joy’ or ‘exclaimed with sorrow’, ‘cry out,
“pray’, etc., according to the sense, i.e.
● Exclaimed with joy–in case of Aha! Ha! Hurrah!
● Exclaimed with sorrow–in case of Ah! Alas!
● Exclaimed with surprise–in case of Oh! What! How!
● Exclaimed with regret–in case of Sorry!
● Exclaimed with contempt–in case of Pooh! Pshaw!
● Applauded with saying–in case of Bravo! Hear!
Use very or great by removing what or how.
Use conjunction ‘that.
Remove exclamatory word and exclamation sign ‘!’ The student must select the verb best suited to the
sense or context, e.g.
● They said, “Hurrah! We have won the match.”
They exclaimed with joy that they had won the match.
● She said, “Alas! I have lost my bridal ring.”
She exclaimed with sorrow that she had lost her bridal ring.
● She said, “How charming the scenery is!”
She exclaimed with surprise that it was a very charming scenery.

Exercises
Select the correct option to report the following sentences.
1. Ritika: Can I borrow your Math book for a couple of days?
Mohit: Yes certainly, I have already studied for the test tomorrow.
Ritika asked Mohit ……………….……………. for a couple of days. Mohit agreed and said that he had
already studied for the test the next day.
a) if she could borrow his Math book
b) to borrow his Math book
c) if could she borrow his Math book
d) if he could borrow her Math book

2.Biology Teacher: I instructed you to draw the diagram of bacteria. Why did you submit a blank sheet?
Sameer: Sir, I had drawn the diagram of bacteria, but you can’t see it because it is not visible to the naked
eye.
The biology teacher had instructed Sameer to draw the diagram of a bacterial cell and asked him
……………………..… a blank sheet. Sameer respectfully answered that he had drawn the diagram but he
could not see it because it is not visible to the naked eye.
a) why he submitted
b) why he had submitted
c) why did he submit
d) why had he submitted

3.Garima: So, after a decade in the industry, are you truly ‘satisfied’?
Karan: I love the film industry. It has its flaws though.
Garima asked Karan if after a decade in the industry he was truly ‘satisfied’
Karan told her ………………….although it had its flaws.
a) that I love the film industry
b) that he loves the film industry
c) that he had loved the film industry
d) that he loved the film industry

4.Neha: I’m really looking forward to the class picnic tomorrow.


Namita: Yes, after a long time we will be meeting our friends and teachers.
Neha told Namita that………..……….. to the class picnic the next day. Namita agreed that after a long
time they will be meeting their friends and teacher.
a) I was really looking forward
b) she is really looking forward
29
c) she was really looking forward
d) I am really looking forward

5. Mother: Would you like to opt for engineering stream after class XII ?
Daughter: I don’t have any interest in engineering.
Mother questioned her daughter if she would like to opt for engineering stream after class XII, then
the daughter replied that ………………………..in engineering.
a) she doesn’t have any interest
b) she hadn’t any interest
c) she didn’t have any interest
d) I didn’t have any interest

Fill in the blanks by choosing the correct option to complete the following sentence.
1. Buddaha in his preaching said that honesty ……… the best policy.
a) was b) has been c) has d) is

2. Piyush asked Mohan if he ……….. lift heavy load.


b) could b) can c) must d)will

3. Mohan said that the government ……… an advisory to the people to wear mask.
a) has issued b) had issued c) have issued d)issues

4. The policeman commanded, “ ……………. the door immediately.”


a) Close b)To close c) Closing d) Closed

5. The teacher said that she ……….. to visit her grandparents.


a) has gone b)went c)had gone d)is going

Do as directed
The class teacher asked Amaan about his upcoming exam. How would Amaan report teacher’s dialogue?
Teacher: How did you prepare for your upcoming exam?
Amaan: I revised all the lessons twice.
The class teacher asked me ………………………

Report Sara’s instruction to Sana


Sara: “ Don’t tear pages from your notebook.”
Sana: Sara advised me ……………………………..
Read the reported sentences carefully. Identify the error and supply the correction in the given format.

1. He told Sita that he has passed the test.


Error Correction

2. My friend told me that he will go to Delhi the next day.


Error Correction

3. I asked my friend if he is happy.


Error Correction
Answers: Reported Speech
Select the correct option to report the following sentences.
1. a) if she could borrow his Math book
2. b) why he had submitted
3. d) that he loved the film industry
4 c) she was really looking forward
5 c) she didn’t have any interest

Fill in the blanks by choosing the correct option to complete the following sentence.
1. d) is
2. a) could
3. b) had issued
4. a) close
5. c)had gone

Do as directed
1. The class teacher asked me how I had prepared for my upcoming exam.
2. Sara advised me not to tear pages from my notebook

Read the reported sentences carefully. Identify the error and supply the correction in the given format.

Error Correction
1. has had

2. Error Correction
will would

3.
Error Correction
is was

Writing Tasks Notes & Worksheets

Formal Letter Writing

Formal letters are official letters written to people in their official capacity or written to a firm. They
are formal in nature. Nothing in such letters should be personal or intimate. with subjects such as inquiry,
orders, transfer of money, reference, request, booking complaints, loss, and apology. What matters is
setting out your letter correctly, keeping to the point, and maintaining the formal tone. These are letters that
are written in a strict and specific format.

Types of Formal Letters

There are different types of formal letters, as discussed, and they can generally be labelled under the
following terms:

1. Business Letters
● Enquiry letter
31
● Complaint Letter
● Order placing letter

2. Official Letters
● Resignation letter
● Complaint letter
● Letter to Principal
● Maternity leave Application
● Job application / cover letter
● Letter to bank manager
● Letter to police
● Letter to government
● Letter to the Editor

An enquiry letter or a letter of inquiry is written to inquire about any product or service. It can also be
written to get information about a course of study. The writer asks for details like price, availability, course
duration, facilities provided, etc.

A Complaint letter can be written to complain against any defective product or defective machinery, asking
for service or replacement. It can also be written to highlight a deficiency in service. A citizen can also
lodge a complaint or grievance via a complaint letter written to the civic authorities.

Order letter can be written by an individual or an organization to place an order of goods. It must contain
necessary details like the description of goods required, quality, mode of payment and expected date of
delivery.

Resignation letter is written by an employee to request termination of services. In case the organization has
a notice period, it has to be complied at both ends.

Letter to the principal can be written by various people for various purposes. A student can write a leave
application to the principal. The parents can also write a leave application for their ward. Parents can write
an application to the principal to get information regarding the course of study. Also, any teacher can write
a letter to the principal, seeking leave.

Maternity leave application can be written by parents – to – be or new parents who have to take care of the
new born child.

Cover letters accompany your application when applying for a new position. The cover letter is a tool to
help introduce yourself in a memorable, personal way during a job application. A well-crafted cover letter
goes over information on your resume and expands this information for the reader, taking them on a guided
journey of some of your greatest career and life achievements.

Letter to bank manager can be written to request for the issue of passbook, issue of new ATM Card,
changes in address, transfer of bank account, renewal of Fixed Deposit, and so on.

Letter to police can be written to raise any matter of concern. It can also be written to report any theft or
loss of valuable possession like passport, driving licence, etc.
Letter to the government authorities like MLA, Governor, Municipal Commissioner, etc. can be written to
raise any complaint or any matter of social concern.

Letter to the editor can also be written to seek help from the publication in highlighting matters of social
concern like traffic jams, lack of amenities, water logging, stray dog menace, plastic use, etc.
Some of the steps in writing formal letters.

1. (i) Introducing oneself if it is the first time you are writing


(ii) Referring to an earlier letter if you are responding to it.

2. Stating the purpose of the letter

3. (i) Stating action/information required from the addressee


(ii) Explaining action taken/supplying information

4. (i) Urging action to be taken


(ii) Offering assistance in future

This is the basic structure of a letter. It will have to be modified according to the purpose for which it is
written and the person to whom it is addressed.

Format of a Formal Letter


1. Sender’s address: Write the sender’s address on the top left side of the page.
2. Date: Leave some space after writing the sender’s address, write the date..
3. Receiver’s address: Then, write the address of the recipient (the Officer / Principal / Editor)
4. Subject of the letter: Mention precisely the main purpose of the letter.
5. Salutation: Write Respected Sir / Madam
6. Body:
Leaving another line, start the body of the letter. The body of any formal letter must be divided into 3
parts:
First Paragraph: Introduce who you are and the purpose of writing the letter in brief.
Second Paragraph: Explain what the issue is and how it impacts you or matters to you and/ or the society.
Third Paragraph: Conclude your letter by mentioning the conclusion or request or solution.
7. Closing: End your letter with complimentary regards and gratitude Use “Thanking you/
/ Yours sincerely”.
8. Sender’s name/signature and designation (if applicable)

Sender’s Address
Date
Receiver’s Address
Subject-
Salutation (Respected Sir/Madam)
——————————————————Body————————————————————-
——————————————————
——————————————————
——————————————————
Complimentary Closing ( Yours sincerely)

Sender’s Name
Designation (if applicable)
33
Candidates writing formal letters in the exam must remember the following tips-
1.
2. Use of colloquial words, short forms, slang language and abbreviations are restricted.
3. The letter must be to the point, precise and clearly indicating the message.
4. The subject line is very important in Formal Letters, even when you reply to a letter.
5. Follow the word limit strictly in order to avoid deduction of marks.
6. Prevent grammatical errors and spelling mistakes to score full marks.
7. Format is of utmost importance.
8. All necessary information must be included in the body of the formal letter.

Formal Letter Writing details


Students shall be expected to attempt any one out of the given two options of formal letter.
The types of letters that can be asked are :
● Letter to the editor
● Letter to the government
● Order letter
● Enquiry Letter
● Complaint letter

The letter has to be written within 100 – 120 words and it has been allotted 5 marks.
The break-up of marks is as follows-

Format: 1
Content: 2
Organization of ideas: 1
Accuracy: 1

Sample Letters

1. Letter to the Editor


You are Christy, a resident of Modi Society, Kolkata. There is an outbreak of the Monkey
virus in your area with a report of 50 new cases. Write a letter to the editor of the local
newspaper India times, highlighting the problem and suggesting measures.

Modi Society
Kolkata

12th June 2023

The Editor
India Times
Kolkata

Subject: Concern about the rising cases of Monkey Virus.

Sir/Madam

Through the columns of your esteemed newspaper, I would like to present my concern about the spread of
diseases like Monkey Virus in our locality.
Now a days many people are suffering from monkey virus because of people coming into physical contact
with an infected animal. This is a highly contagious infectious disease and if timely action is not taken, it
will spread magnanimously.
Seeing the situation of the people in our locality, I request the government to take action.
Animals should be treated with proper medication and those infected should be quarantined.
I hope you will share my request in your reputable newspaper as soon as possible, alerting the
government about the situation in the area.

Thanking You

Yours sincerely

Christy

2.Letter to the government


You are Rohan/Rohini, the resident of Vikas Nagar, Hyderabad. You have observed an
increase in road accidents, making you and other residents fearful. Write a letter to the Municipal
Commissioner regarding this issue.
#355
Vikas Nagar
Hyderabad

7 July, 2023

The Municipal Commissioner


Hyderabad

Subject: to express my concern on increasing road accidents

Respected Sir/Madam

There has been an increase in road and car accidents in Vikas Nagar since 1 July. I wish to draw your
attention towards this issue.
Road accidents are often caused by the carelessness of drivers. However, in this case, the major contributor
is not just the irresponsible drivers but the potholes on the road. The roads have become unpaved with time.
In addition to this, the traffic lights do not function properly and when they stop working, the traffic police
do not come immediately which causes traffic jams, road accidents and chaos.
Kindly look into this matter as soon as possible as people are becoming increasingly fearful while driving.

Yours Sincerely
Rohan/Rohini

(Resident of Vikas Nagar)

3. Letter for placing Order


You are Laxmi of Bright Stars Senior Secondary School, Ghaziabad. Write a letter to the Sales
Manager of The Books Point, Ghaziabad placing an order for some books for your school library.
Bright Star Senior Secondary School
Ghaziabad

15 July 20XX
The Sales Manager
35
The Books Point
Ghaziabad

Subject: Supply of books for the School Library


I am writing this letter in reference to the quotations that you sent for the books we asked for. I want to tell
you that we intend to buy those books from you on the quoted amount by you. Please send the following
books by the end of this week. The quantity of each book is mentioned against the books in the list.

Name Author Quantity


1. As you like it Shakespeare 20
2. Great Expectations Charles Dickens 30
3. Timeline Michael Crichton 20

You are requested to send the above books as per the terms and conditions. Kindly make sure the books to
be in good condition, well–bound and packed properly. You are also requested to send the bill along with
the books after applying the discount permissible to schools. Payment will be made soon after the receipt
and checking of the books.
Damaged books will not be accepted nor any payment will be made for the same.

Thanking you
Yours sincerely
Laxmi

4. Letter of Enquiry

Advertisement

Institute of Hospitality Management


F-5, Junk Road, New Delhi
A few seats available in Housekeeping course.
Contact immediately for further details.

You are Anu, 12A, Fort Road, Agar. You read the above advertisement in a local daily. You want to
pursue the course in housekeeping. Write a letter to the Director of the Institute inquiring about the
duration of the course, fees other details. The letter should be in 100-120 words.

12 A, Fort Road
Agra

15th July, 20XX


The Director
Institute of Hospitality Management
F-5, Junk Road
New Delhi

Subject: Enquiry about Housekeeping Course


Sir,
This refers to your advertisement in the newspaper regarding the Housekeeping Course being conducted by
your institute. As I want to pursue the course, I want to know about the duration of the course, the fees to
be given, the date of commencement and the syllabus to be covered. I have completed my graduation in
Home Science from Agra University and want to pursue a career in the hotel industry. Further, I want to
enquire whether you provide hostel facility for outstation girl candidates. As I do not live in Delhi. Also,
please inform me the charges for the hostel accommodation.

Thanking you
Yours sincerely
Anu

5. Complaint letter (civic authorities)

You are Raghav/Renu Sharma, President, RWA, C Block, Janak Puri, New Delhi. The main park in
your locality which was once a model park lies in a state of utter neglect. It has become a favourite
spot for anti–social elements and a permanent resting place for stray animals. Write a letter to the
Municipal Commissioner requesting him to instruct the horticulture department to take prompt
action and restore the park to its original condition.

RWA, C Block,
Janak Puri
New Delhi 1100XX

17 August 20XX
The Municipal Commissioner
New Delhi 1100XX

Dear Sir/Madam

Subject: Poor condition of the central park in C Block, Janak Puri

Through this letter I would like to draw your attention to the miserable condition of the central park in C
Block area. Once, a model park, it now lies in a state of utter neglect. It has become a favourite spot for
anti–social elements and a permanent resting place for stray animals. There is no greenery left in the park.
People use this park to gamble and take drugs. There is no place left for children to play and old people to
take a walk. The Residents’ Welfare Association (RWA) has left no stone unturned to do their bit in
improving the condition of the park but all in vain.
These activities are causing a lot of problems to the residents and spoiling the atmosphere of the locality. I
request you to take immediate remedial action to restore the beauty of the park.
Thanking You
Yours sincerely
Renu Sharma

6. Complaint letter (company/service provider)

You are Raman/Reena of 240, Shahid Nagar, Agra. Write a letter to M/s Raju Sales Corporation
complaining about the poor performance of the washing machine purchased from their showroom,
mentioning clearly the specific problem you are facing and the action you want. Give all the relevant
details.

37
240, Shahid Nagar
Agra
25 January 20XX
The Manager
M/s Raju Sales Corporation
Agra

Subject: Complaint about defective washing machine

Dear Sir
I bought an LG washing machine from you Shahid Nagar Branch on 1 January 20XX vide cash memo
number 123. It has a warranty of two years. I regret to inform you that it has developed a technical snag in
the first month itself. Its water dispenser is not working properly. The dryer and water outlet are choked
and sometimes it gives electric shock too. It’s a matter of grave concern that a brand new machine should
develop so many defects in the first month of its purchase.
It is not only disappointing but shocking too as the electric shocks could put the user’s life at risk. Since it
is still under warranty, you are, therefore, requested to replace it at the earliest. I sincerely hope that such
cases are few and far between and do not get repeated.
Yours faithfully
Raman

Writing Tasks-Worksheet #1

Q1. People have adapted to wearing face masks as a ‘new normal’. But most of them do not know how to
dispose of them properly. Spread awareness regarding the disposal of protective masks by writing a letter to
an editor of a popular newspaper.

Q2. You live in Saharanpur, Uttar Pradesh. In your locality, the street lights are not well functioning and thus
it gets really dark in the evening. Write a letter to the Municipal Corporation of your city regarding this
problem.

Q3. You are the Head of the Department of Mathematics in Jesus & Mary School, New Delhi. Write a letter
to the Sales Manager, Cambridge University Press, New Delhi, requesting them, a list of Magazines,
Newsletters and Journals of Mathematics to send to the head of the department. You are Dr. D.K. Singh.

Q4. As a health conscious person, you have noticed an advertisement in a newspaper on yoga classes in your
neighbourhood. Write a letter in 100 120 words to the Organiser, Yoga for Public, R.K. Puram, New Delhi
requesting him/her to send you information about the duration of the course and other relevant details. You
are Shweta/Srikar of 15, R.K. Puram, New Delhi .

Q5. You are Shubham/Subhi Tyagi of 1, Ashok Road, Lucknow. You are deeply concerned about the
increasing number of road accidents in your area. You strongly feel that reckless driving by the youngsters
is the primary cause of such accidents. Write a letter to the Traffic Police Commissioner requesting him to
take prompt and appropriate action. Give concrete suggestions to deal with the problem.

Q6. You are the Physical Education Instructor at St. Jesus and Mary School, Agra. You had placed an order
for sports goods with Student Sports Enterprises. When the items arrived you found that some of them were
defective. Write a letter to the Manager, Student Sports Enterprises, asking him to replace the defective
items.

ANALYTICAL PARAGRAPH WRITING


An analytical paragraph is a form of descriptive writing which is written on the basis of a given chart,
graph, data, outline, clues, table etc. It requires analysis and comparison of the given facts and drawing
conclusion based on the given data.

SOME SIGNIFICANT FEATURES OF AN ANALYTICAL PARAGRAPH ARE:

● It describes the given chart, graph, table, data, clues etc.


● It should be short and informative.
● It should clearly mention the provided facts and figures.
● The whole paragraph should be in the same tense.
● The language used should be simple and accurate.
● No personal observation or feedback should be added.

TYPES OF ANALYTICAL PARAGRAPHS

◆ PROBLEM AND SOLUTION BASED:


A problem is presented and learners are expected to suggest some solution based on some logic. They are
supposed to support their solution with arguments, reasons and/or data.

◆ DATA BASED:
Some data is presented in the form of a pie-chart, a bar graph, a line graph, a table or a combination of any
two of these and learners are expected to analyse the data making comparisons and drawing conclusions and
to summarise it in the form of a paragraph.

◆ INFORMATION BASED:
Some information is presented as an input and students are required to process and understand that
information and then summarise it in a paragraph. Sometimes, they may also be required to give their own
opinions or draw their own conclusions, too.

◆ PROCESS DESCRIPTION BASED:


A flow chart, a diagram, a visual or an infographic is presented and students have to analyse the information
it presents, usually a process showing the sequence or different stages of action in the making of something.

◆ MAP BASED:
Usually a set of two maps is presented as an input with significant differences in a piece of land, a landscape
or a building plan showing some changes, additions, and alterations therein. Students are supposed to
carefully analyse these changes and summarise them in the form of a paragraph.

◆ BEHAVIOUR,FUNCTION,PATTERNORCYCLEBASED:
A flow chart, a diagram, a visual or an infographic is presented as in a process and students have to analyse
and summarise the information it presents, usually the behaviour or pattern or structure of an organism or
various traits of the character or profile of a professional or the life cycle of a species etc.

◆ SIMILARITIES AND DIFFERENCES BASED:


A set of two visuals or diagrams or infographics is presented with one showing marked differences and/or
similarities in comparison with the other. Students are required to notice and mark those differences and
similarities and then summarise them objectively in the form of a paragraph.

◆ SUGGESTION OR ADVICE BASED:


39
A situation is presented and students are asked to put forth their suggestions or advice on what the person(s)or
organisation(s)involved in it should do.This type is similar to the problem and solution based type of
analytical paragraphs.

SOME USEFUL EXPRESSIONS FOR WRITING AN ANALYTICAL PARAGRAPH


To report an increase
◆ Has risen/increased/gone up/shot up…
◆ Has experienced/seen a rise/an increase in…
◆ Has shown/registered a rise/an increase…
◆ Has reached a maximum/a peak of…
To report a decrease
◆ Has fallen/gone down/come down to…
◆ Has dropped from......… to .......…
◆ Has experienced/registered/shown a fall/a decrease/a decline in….
◆ Has hit/touched/reached its lowest/minimum level/point...
To report differences
◆ Is better than/more than double/half as much/three times more/less than....
◆ Not as big as/as much as/as large as...
◆ Many times over/more/less...
To report similarity
◆ Is the same as/similar to...
◆ Is/Are comparable with...
◆ Can be grouped together/classified together/clubbed with...
◆ Show(s) similarity/similarities...
◆ There are striking similarities between...
To report comparisons and contrast
◆ As compared to…
◆ In (sharp) contrast to/with…
◆ Is comparable/not comparable with…
◆ Compares well with.../Contrasts with…
To report constancy or no change
◆ Has seen/registered/experienced/shown no change...
◆ Has remained constant/unchanged/the same...

Frequently Used Tenses


Present Perfect: Example – The number of accidents has risen sharply.
Simple Past: Example – Fewer cases of theft were reported; the sales rose up by 30 per cent. Will + base
verb: Example – At this rate the population will touch 1.5 billion mark by 2022. Will + have + past
Example – At this rate the population will have touched 1.5 billion by the
participle verb : year 2022.
Passive Structures: Example – A few other alterations were also carried out in the hotel.

LAYOUT OF AN ANALYTICAL PARAGRAPH


Given below are the results of a phone survey of a group of 1000 students aged 13-16 carried out by an
education company to find out how effective online classes are. Study the data carefully and then write an
analytical paragraph to summarise the information. Make comparisons wherever appropriate and draw your
own conclusions
SAMPLE ANALYTICAL PARAGRAPH (PROBLEM & SOLUTION BASED)
You are the General Manager of a restaurant. You have noticed that the cost of seafood items has gone up in
recent times and this has overshot the budgets of raw materials procurement. Your supplier explains that the
supply chain of seafood items is adversely affected due to bad weather and a disruption in transport due to a
strike, and consequently the prices of seafood items have gone up.

Write an analytical paragraph briefly describing how the cost of procuring seafood items has overshot
the procurement budget of the restaurant and suggesting a solution to the problem in consultation with
41
the restaurant chef and sales and marketing manager such that the seafood continues to be on the
menu but the restaurant does not incur any losses or lose valuable customers to the competition.You
might also like to introduce a new menu with attractive alternatives to seafood.

The restaurant is finding it increasingly difficult these days to serve its patrons a range of seafood items like
prawns, shrimps, crabs, lobsters, oysters and tuna fish. While this may have left many of our customers
disappointed, the restaurant is unable to please them by overshooting its monthly budgets of raw materials
for sea-food dishes. Our vendor Messrs Bombay Seafood Suppliers have expressed their inability to provide
these items at the existing rates owing to the disruption caused in their procurement due to inclement weather
and a transport strike. Faced with this predicament, the restaurant has the following options – one, increase
the prices of sea-food based dishes; two, offer smaller portions to save on raw materials; and three, take away
our customer's interest in sea- food by introducing a fortnight long food festival with a new menu of non-
vegetarian items based on chicken, mutton, lamb and fresh water fish and eggs as well as a whole new range
of Indian and Chinese vegetarian menu, including regular South Indian delicacies. This will not only help the
restaurant save on its raw materials supply expenses but also keep our customers happy with a variety of
alternative food menu. This matter was discussed with the Chief Chef as well as the Food and Beverages
Manager, who are aligned to the proposal spelt out above. Subject to the management approval, the new food
festival can go live next weekend.

SAMPLE ANALYTICAL PARAGRAPH (DATA BASED)

Given below is a bar graph showing some changes in the public behaviour in 'X' city in a period of
two years.Analyse the data carefully and then summarise it in the form of an analytical paragraph.
Writing Tasks-Worksheet # 2

Q1. Your school has 4 buses each with a capacity of 50 students. The total number of students who actually
travel by these buses is 175. Another 150 students have applied for school transport. The school is not in a
position to buy new school buses.
What should school authorities do to ensure that school buses run to full capacity and the students in the
waiting list for school transport get it.
Write an analytical paragraph in about 150 words to briefly describe and to suggest some practical or viable
solutions to the problem.

Q2. The bar graph given here, depicts the adult literacy rate in two states of India. Write an analytical
paragraph in 100 120 words drawing a comparison between the two and write possible reasons for this
difference.

43
Q3. Negligent disposal of single-use plastic bags leads to choked drains. Single-use plastic is non-
biodegradable and causes dangerous substances to leach into the water and soil. It should be banned
completely. Write an analytical paragraph in 100 120 words on the above given input.

Q4.The following table shows details about the different Internet activities for six categories of people
according to different age groups. Study the table and write an analytical paragraph in about 100 120
words, drawing comparisons between the different purposes. Use the given input.

Q5. It is distressing to see the attitude of people towards animals. The number of cases of cruelty towards
animals is increasing. The following pie chart represents the attitude of people towards animals. Write an
analytical paragraph in 100 120 words interpreting the data given in the pie chart.
LITERATURE SECTION

Short Questions.
1. Do you think that Lencho was right to call the post office employees a bunch of crooks? Why or why
not?
2. Why did Lencho write a letter to God? [Letter to God]
3. What promise does Mandela make at the beginning of the opening of his oath-taking speech?
4. What did Mandela realise about his brothers and sisters? [Mandela]
5. How did the young seagull and his family celebrate his first flight?
6. What did the writer feel inside the clouds? [Two Stories about Flying]
7. Describe your views about Mr Keesing as a teacher.
8. Why does Anne Frank think that ‘paper’ has more patience than ‘people’? [Anne Frank]
9. Why was the Baker’s furnace essential in a traditional Goan village?
10. Describe Coorg’s weather. When is it most pleasant for the tourists to visit Coorg?
11. Why did Pranjol’s father say that Rajvir had done his homework before visiting Assam? [Glimpses of
India]
12. What, according to the writer, is the ‘real play’ of the otter?
13. How did Mij behave outside the house in London? [Mijbil, the Otter]
14. According to Kisa Gotami what is the greatest grief of life?
15. What was the effect of the suffering of the world on Buddha? [Sermon at Benares]
16. How did Valli save money to travel by bus?
17. What was the most fascinating thing that Valli saw on the street? [Madam Rides the Bus]
18. How does Natalya excite Lomov to the point of verbal fighting?
19. How does Lomov react when Chubukov says that he is not used to misbehaviour by a young man like
Lomov? [The Proposal]
20. How has Frost brought out the contrasting ideas in the poem?
21. How does Robert Frost caution the common man? [Fire and Ice]
22. Why do you think the tiger looks at the stars?
23. How does a tiger create terror for the villagers? [A Tiger in the Zoo]
24. Write the sum and substance of the poem, “How to Tell Wild Animals”.
25. What is so weird about the Hyena and the crocodile? [How To Wild Animals]
26. Money is external”. What does the poet mean by this expression?
27. What does John Berryman want to convey through this poem? [The Ball Poem]
28. What is the central theme of the poem Amanda?
29. Why does Amanda wish to be a mermaid, an orphan, or Rapunzel? [Amanda]
30. Difficulties come but they are not to stay forever. They come and go. Comment referring to the poem
‘Fog’.
31. How does the poet compare fog to a living being? [Fog]
32. What did Custard look like?
33. But Custard cried for a nice safe cage. ’ Who is Custard? Why did he cry for a ‘nice safe cage’? [The
Tale of Custard Dragon]
34. What are Anne’s views on external beauty?
35. Why is only God capable of loving Anne for herself? [For Anne Gregory]
36. What is a “dust of snow”? What does the poet say has changed his mood? How has the poet’s mood
changed?
37. What is ‘a hemlock tree’? Why doesn’t the poet write about more ‘beautiful’ trees such as a maple, or
an oak, or a pine? [Dust of Snow]
38. What are the three things that can't happen in a treeless forest?
45
39. The poem "The Trees' presents a conflict between Man and Nature. Discuss the statement. [ Trees]
40. Hari Singh did not catch the train deliberately but he could catch it easily. Why?
41. Why was Hari Singh grateful to Anil? [The Thief Story]
42. Why was Mr. Harriot tempted to keep Tricki as a permanent guest?
43. How would you describe the behaviour of the rich woman Mrs Pumphrey, pampering and overfeeding
Tricky? [A Triumph of Surgery]
44. Give the reasons which made Horace Danby commit a theft at Shotover Grange.
45. Horace Danby gets annoyed when anyone talks about ‘Honour among thieves’. Comment. [A Question
of Trust ]
46. How did Ebright’s mother help him?
47. “But there was one thing I could do-collect things.” What collection did Ebright make? When did he
start making the collection? [The Making of Scientist]
48. Why did Ausable frame the story of the balcony?
49. Who was Max? How did Ausable get rid of him? [The Midnight Visitor ]
50. What was Loisel’s reaction to his wife’s desire for a new dress?
51. Why was Matilda Loisel always unhappy? [Necklace]
52. Does she find her teacher different from the people at home?
53. Why did Bholi refuse to marry Bishamber at last? [Bholi]
54. Why was the twentieth century called the ‘era of books’?
55. Why are books referred to as a man’s best companion? Which book saved the Earth from Martians in
invasion? [The Book that Saved the Earth]

Answers:
1. Lencho called the post office employees a bunch of crooks as he did not get the full money he had
demanded. He could not believe that God had sent him any less money so he doubted these people. But he
was not right to call them a bunch of crooks.

2. When the hailstorm destroyed Lencho’s crops, he wrote a letter to God because he was the only hope in his
despair. Lencho asked him to send a hundred pesos to sow his field again and support his family.

3. Mandela thanks all the international leaders and guests as he calls it an occasion of joy and victory for
Justice. He promises that the country shall not again experience the oppression of one by another.

4. Mandela realised his brothers and sisters were not free in their own country due to their colour. The
freedom of everyone in his society was curtailed. He joined the African National Congress and fought for
freedom.

5. When the young seagull started flying and got over his fear, his family screamed around him out of joy.
They praised him and offered him scraps of dogfish out of delight as he made a successful attempt.

6. When the writer entered the clouds, it became impossible to see outside the aeroplane. The aeroplane
jumped and twisted in the air and all the instruments like compass etc stopped working due to the weather
conditions.

7. Mr Keesing seems to be a strict teacher but he believes in taking actions with good intentions and for the
development of children. He tries to control her bad habit of talking too much but as soon as he is
convinced that her habit does not affect her studies, he overlooks her shortcomings.

8. Anne believes that paper has more patience than people because it listens to her more patiently and silently
it does not react like other people and also because she can confide in her diary all her secrets.
9. Different kinds of breads are important on different occasions. Bolinhas had to be prepared during
Christmas and other festivals. The mothers used to prepare sandwiches on the occasion of their daughter’s
engagement. So, the baker’s furnace was essential.

10. The weather of Coorg is pleasant during the months from September to March. During that time, the
weather is perfect with some showers thrown in for good measure. During the monsoon, it receives heavy
rainfall.

11. Rajvir was very excited about visiting the tea garden and thus, he studied a lot about it before visiting the
tea garden. Thus, Pranjol’s father said that Rajvir had already done his homework before visiting Assam.

12. The real play of an otter is lying on his back and juggling with small objects between his paws. Mijbil
would roll two or more marbles up and down on his wide, fat belly. He never dropped one to the floor.

13. Mij used to follow the narrator while walking on the streets. He developed certain habits during those
walks. He went with him like children playing, running and touching things in the street. He would take the
author near the wall of a primary school and then gallop on it.
14. According to Kisa Gotami, the greatest grief in life is the death of one’s loved ones and one’s inability to
stop them from dying. Therefore, instead of lamenting on it, the wise should not grieve. Weeping will only
increase the pain and disturb the peace of mind of a person.

15. At the age of 25, while hunting, one day Buddha saw a sick man, then an aged man, then a funeral
procession and finally a monk begging for alms. These moved him so much that he went out into the world
to seek enlightenment.

16. Valli had thriftily saved whatever stray coins came her way. She resisted every temptation to buy
peppermints, toys, and balloons and ride on the merry-go-round at the village fair to save money for her
bus journey.

17. The most fascinating thing that Valli saw on the street was the bus that travelled between her village and
the nearest town. It passed through her street each hour, once going to the town and once coming back.

18. Natalya repeatedly insisted that Oxen Meadows are theirs and told Lomov that upto now she considered
Lomov as a good neighbour and friend. This excited Lomov to the point of verbal fighting.

19. Lomov reacts by saying that Chubukov thinks that he is a fool. He says that he cannot talk to him calmly
and politely as he is making a false claim to his property. He further accuses Chubukov of being a grabber.

20. ‘Desire’ propels us in hot pursuit of something, hence, it is compared with fire. ‘Hatred’ makes us cold
towards other’s feelings. The coldness of ice can numb our senses. Similarly, the coldness of our hearts can
numb our kindness. That is why ‘hatred’ has been compared with ice.
21. Man is selfish, avaricious, lustful, indifferent and hateful. Robert Frost cautions the common man and says
that he should not forget the bitter reality that everything in this world is transitory and death is inevitable.
22. The tiger feels very helpless in the cage. He stares with hope at the brilliant stars shining in the sky. He
hopes for the day when he will be able to run free in the wild. The brilliant stars, thus, provide him with
some sort of comfort.
23. The tiger creates terror for the villagers by snarling around their houses as they are situated near the jungle.
He frightens them by showing his white fangs and claws.

47
24. The poet, Carolyn Wells, in the poem suggests some dangerous ways to identify wild animals. The poem is
full of humorous examples when the poet tries to distinguish one animal from the other. Moreover, the
poem educates us by describing the various features of wild animals.
25. Some animals such as the hyena and the crocodile are famous for their weird behaviour. For example, a
laughing hyena’s voice resembles a human’s laughing sound. Moreover, a hyena laughs while swallowing
its prey, while a crocodile sheds tears.
26.The poet makes the boy understand about his responsibility as the loss is immaterial. Money is external
as it cannot buy memories, nor can it replace the things that we love, the things that matter.

27.Poet, John Berryman wants to convey the importance of loss and responsibility in life. We all should
learn our responsibility and how to cope with the loss.

28.The poem Amanda by Robin Klein highlights the importance of the upbringing of a child. It points out
that upbringing doesn’t involve making a child responsible and fit for society only. It is important to note
that upbringing involves understanding from both sides. One cannot just force a child to be civilised and
good-mannered. “Love and proper care are required in nurturing a child.

29.Amanda wishes to be a mermaid so she can drift alone by the blissfully languid, emerald sea. She yearns
to be an orphan so that she is able to roam the sea and make patterns using her bare feet. Being Rapunzel
means she could live carefree on a high tower. Amanda wishes to be there so that she can avoid her
suffocating reality.

30.The poet compares the fog to a cat. The silent steps of a cat and the way it sits on its haunches are very
similar to the way fog comes and surrounds the city and looks over it.

31.Difficulties, when faced by people, tend to leave them hopeless and shattered. It takes a lot of courage to
overcome any problem and to solve it. If we take a clue from the poem and compare difficulties to fog, we
find that just like fog, difficulties also come and go. [One need not be hopeless and lose courage when
problems come, one should rather think of it as fog, meaning that it has not come to stay but will always
leave, like fog.]

32.Custard looked dangerous with spikes on his top and scales underneath. His mouth was like a fireplace
and his nose was like a chimney. His toes looked like daggers.

33.Custard is Belinda’s pet Dragon. He cried for a nice safe cage because he was a coward, who feared
easily and looked for comfort and safety of himself.

34.Anne does not believe in external beauty and wants to be loved for herself. She says that her beautiful
hair which attracts so many men can be changed and coloured differently. This shows that Anne thinks that
external beauty is not important and it is changeable.

35.Only God is capable of loving Anne for herself because humans do not have the power to look at the
beauty of the soul. Only God is capable of not caring for the outer beauty, looking at the beauty of one’s
soul.

36.The ‘dust of snow’ means the fine particles or flakes of snow. The sudden shower in the form of the dust
of snow changed the poet’s mood. The poet’s mood changed from sad to happy. He felt refreshed and
wanted to enjoy the rest of the day.

37.A hemlock tree is a poisonous plant with small white flowers. The poet, Robert Frost, didn’t choose to
use an oak, maple or pine tree. Instead, he chose the hemlock tree and left all the beautiful trees present in
the world. He did so to present his mood and feelings.
38.The three things that can't happen in a treeless forest are : (i) the sitting of a bird on trees, (üi) the hiding
of insects and (üi) the sun burying its feet in the shadow of the forest.

39.The poem "The Trees' presents the rebellion of the tree against human oppression and imprisonment
within walls. The forest is the natural habitat of the trees. The trees feel suffocated in houses. They rebel
against it and move out.

40.Hari Singh did not catch the train deliberately because his inner conscience held him back. He thought
that Anil would feel sad not for the loss of money but for the loss of trust he had reposed in him. Moreover,
he did not want to lose the chance of learning. His inner self was transformed.

41.Hari Singh was grateful to Anil because Anil taught him to read and write whole sentences and to add
numbers. Soon he would become an educated man and would be able to achieve anything in life.

42.Mrs. Pumphrey used to send lots of things like eggs, wine, brandy etc., for Tricki. But nothing was
given to Tricki. All the things were consumed by the doctor and the other members of the hospital. Hence,
Mr. Harriot was tempted to keep Tricky as his permanent guest.

43.Mrs. Pumphrey behaved in a very silly and thoughtless manner by over-pampering Tricky. Indirectly,
she was responsible for his ill health. It was a waste of money. But this behaviour is very common among
rich people. It is a general problem for those who live alone.

44.Horace chose Shotover Grange because it had a safe, and consisted fifteen thousand pounds worth of
jewels. He robbed a safe only once a year and stole an amount that would last for one year and nothing
more. He also had the building plan of the house. Horace was in strong need of money to buy new books as
he was a great reader of rare books.

45. Horace Danby was a good and respectable citizen but he was not completely honest. He had a strong
passion for books. So he robbed once a year only from the rich to buy books. On one of such attempt, he
was fooled by a lady thief who was very clever and mean. Horace realized this fact only when he was
arrested for the robbery.

46. Ebright’s mother was a great help to him. She always encouraged his interest in learning and finding
more. She took him out on trips. She also bought him a telescope, a microscope, cameras, mounting
materials and other equipment that helped him in many ways.

47. When young, Ebright felt he could neither play football nor baseball but there was one thing he could
do and that was collecting things. And then he started collecting things. He had a wonderful assortment of
monarch butterflies, fossils, rocks and coins.

48. Ausable knew that Max was scared and he used his ignorance and foolishness in his favour. He framed
the story of the balcony to convince max about the balcony which was non-existent. It helped Ausable to
get out of a dangerous and critical situation.

49. Max was a secret agent, who came for secret papers on missiles. Ausable outwitted him by making a
false story about the balcony. Max tried to escape through it and died.

49
50. The Loisel’s had been invited to a party at the minister’s residence. When Matilda told her
husband that she wanted a new dress to wear, first of all he asked her to wear whatever she possessed.
When she got angry and refused to go to the party at all, he finally agreed to let her buy a new one.

51. Matilda Loisel was very pretty. She considered herself being born for a luxurious life but the reality
differed from her dreams. In reality she was married to a petty clerk and led a very simple life. Her fantasy
over ambitiousness made her unhappy.
52. Bholi found her school teacher to be different from her family members because the teacher was a kind
and considerate lady. She had a soft and soothing tone. She gave Bholi a lot of encouragement and
inspiration which she was not getting from her family.

53. In the end, Bholi refuses to marry Bishamber because he was greedy and mean. She refused to allow
him to garland her and told him that he was a contemptible person.

54. The twentieth century was called the ‘era of books’ because they were the encyclopaedias of
knowledge. There were books for everything—from ant-eaters to Zulus. Books were used to educate
people and teach them. In fact, books were a very important part of life of the people.

55. Books have been called a man’s best companion as they always teach him the right things. They are the
best companions because your friends may desert you in your hour of need but a book will never leave you
or give you the wrong guidance.

REFERENCE TO CONTEXT:

1. When he finished, he went to the window to buy a stamp which he licked


and then affixed to the envelope with a blow of his fist. The moment the
letter fell into the mailbox the postmaster went to open it. It said: “God: Of
the money that I asked for, only seventy pesos reached me. Send me the
rest, since I need it very much. But don’t send it to me through the mail
because the post office employees are a bunch of crooks. Lencho.”

i. ‘…bunch of crooks.’ Pick the option that DOES NOT collate with ‘bunch of’,
correctly. (1m)

a) option (i)
b) option (ii)
c) option (iii)
d) option (iv)
ii. Pick the option that lists the option corresponding to—'with a blow of his
fist.’ (1m)

a) Option (i)
b) Option (ii)
c) Option (iii)
d) Option (iv)

iii. Pick the most suitable quote for this extract. (1m)
a) “It is easier to fool people than to convince them that they have been
fooled.” – Mark Twain
b) “Real knowledge is to know the extent of one’s ignorance.”- Confucius
c) “You see a person’s true colours when you are no longer beneficial to
their life.”- anonymous
d) “True generosity means accepting ingratitude.” - Coco Chanel

iv. What did Lencho’s letter include? What was in Lencho’s mind when he was writing the letter ? (2m)

Answers: i)d ii) d iii) d iv. List of further demands. Lencho thought that only God could help him in that
hour of need.

2. The way a crow


Shook down on me
The dust of snow
From a hemlock tree
Has given my heart
A change of mood
And saved some part Of a day I had rued.
(Dust of Snow)

i. Of the many symbols the hemlock tree represents, choose the one that Frost drew upon in all likelihood,
for this poem. (1m)

Symbol of
a) longevity.
b) togetherness.
c) healing.
d) protection.
51
ii. Choose the option that lists the possible feelings of the poet before the experience shared in the poem.
(1m)

1) reassured
2) disappointed
3) curious
4) demotivated
5) thankful
6) disheartened
7) impulsive

a) 1, 3 &7
b) 2, 4 & 6
c) 5&7
d) 1&3

iii. Identify the option that DOES NOT use the word ‘rue’ correctly. (1m)

a) The film was a disaster and he rued his decision to act in it.
b) I am sure she rued the day she listened to a fortune-teller.
c) It wasn’t long before I rued my disobedience and my deceit.
d) Others finally rue the one who is dishonest and heartless.

iv. Name the poet. What does the poet say has changed his mood? (2m)

Answer:
i) c ii) b iii) d iv) Robert Frost. The flakes of snow were dropped by the crow which changed the poet’s
mood.

3. Some say the world will end in fire, Some say in ice.
From what I’ve tasted of desire I hold with
those who favor fire.
But if it had to perish twice,

I think I know enough of hate


To say that for destruction ice
Is also great
And would suffice. (Fire and Ice)

i. Select the option that correctly classifies the connotations for fire and ice, as suggested in the poem.

(1) rage (2) violence (3) indifference (4) hate (5) greed

a) Fire- 3,4; Ice- 1,2,5


b) Fire- 2, 5; Ice-1,3,4
c) Fire-1,3,5; Ice- 2, 4
d) Fire- 1,2,4; Ice- 3,5
ii. The poem is a _________, put across by the poet.

a) powerful warning
b) heartfelt apology
c) earnest appeal
d) vengeful threat

iii. Pick the option that is NOT TRUE about the poet according to the extract.

The poet
a) is inclined to believe that the world would most likely end with fire.
b) has heard divided opinions about the way the world would end in all likelihood.
preaches love and kindness to combat the spread of hate among

iv. Identify the tone of the poet in the line ‘To say that for destruction ice/Is also great’. How are ice and
fire similar to each other though they have contradictory traits? (2m)

Answers: i) d ii) a iii) d iv) Sarcastic. Both ice and fire are similar in the in the sense that both of them
would destroy everything in the world.

4. We, who were outlaws not so long ago, have today been given the rare privilege to be host to the
nations if the world on our own soil. We thank all of our distinguished international guests for having
come to take possession with the people of our country of what is, after all, a common victory for
justice, for peace, for human dignity.” (Nelson Mandela: Long walk to Freedom)

i. The guests at the spectacular ceremony are being called distinguished because they (1m)

a) they have been invited as guests to attend it.


b) they are eminent world leaders witnessing it.
c) they are visiting the country for this purpose.
d) they have resumed diplomatic relations with the country.

ii. It is a victory for ‘human dignity’. Pick the option that lists the correct answer for what ‘human dignity’
would include. (1m)

a) i) equality ii) liberty iii) indecency


b) i) liberty ii) indecency iii) self-respect
c) i) immorality ii) self-respect iii) equality
d) i) equality ii) liberty iii) self-respect

iii. Why does the speaker say that it is a ‘rare privilege’? (1m)

He says this as they have


a) been deprived of this honour.

53
b) seldom been given this honour.
c) experienced it for the first time.
d) been chosen over other countries, for this honour.

iv. Who were the ‘outlaws’? What did the people of South Africa achieve at last? (2m)

Answers: i) b ii) d iii) c iv)Nelson Mandela, on his inauguration ceremony, mentions the nation of
South Africa as outlawsThe people of South Africa achieved their political emancipation at last.

5. He should be lurking in shadow,


Sliding through long grass Near the
water hole Where plump deer pass.

(A Tiger in the Zoo)


i. According to the extract, the poet wishes for the tiger to be ‘sliding’ through the foliage as this would

a) assist in keeping the prey unsuspecting of the predator’s sound.


b) aid in camouflaging the presence of the predator before it rushes in.
c) help the predator pounce on the prey comfortably without getting tired.
d) Support the predator’s vision as it eyes its prey. (1m)

ii. Which fact DOES NOT connect with the significance of the water hole for the tiger? (1m)

a) Many tigers chase prey into the water and hold the victim’s head underwater until it drowns.
b) Prey feed in the water on water-lilies, and often wander into the middle of the water hole, where they
are vulnerable and easy for the tiger to kill.
c) Prey which has quenched its thirst ensures the consumption of hydrated meat for the tiger.
d) Chasing the panicked prey from shallow to deep water where the tiger grabs it.

iii. Pick the phrase that DOES NOT suggest that the forest in the extract is lush.
a) long grass
b) the water hole
c) plump deer
d) lurking in shadow
‘shadow’ here, refers to the shadow of (1m)

a) the tiger.
b) long grass.
c) water hole.
d) deer

iv. What does shadow refer to? Who would pass through the water hole? (2m)

Answers: i)a ii) c iii)d iv) Long grass. A plump deer would pass through the water hole.

6. He just felt a bit dizzy. Then he flapped his wings once and he soared upwards. “Ga, ga, ga, Ga, ga,
ga, Gaw-col-ah,” his mother swooped past him, her wings making a loud noise. He answered her with
another scream. Then his father flew over him screaming. He saw his two brothers and his sister flying
around him curveting and banking and soaring and diving. Then he completely forgot that he had not
always been able to fly, and commended himself to dive and soar and curve, shrieking shrilly.
(Two Stories about Flying)

i. Which of the following mirrors the use of the literary device in “shrieking
shrilly”?
a) sparkling saga
b) singing soft
c) slippery sloppily
d) sneeze silently

ii. The line “he completely forgot that he had not always been able to fly”
implies the
a) great confidence the young gull had in his skills.
b) naturalness of the act of flying for the young gull.
c) satisfaction and joy of flying together as a family.
d) desire of the young gull to leave his fears behind.

iii. The extract refers to the many movements of the young gull’s brothers and sister. Choose the
option that correctly sequences these movements. (1m)

a) The young gull’s brothers and sister flew by tilting their wings, rose high, made darting movements
and plunged headfirst.
b) The young gull’s brothers and sister flew by plunging headfirst, making darting movements, titled
their wings and rose high.
c) The young gull’s brothers and sister flew with darting movements, titled their wings, rose high and
plunged headfirst.
d) The young gull’s brothers and sister flew by rising high, plunging headfirst, making darting
movements and tilting their wings.

iv. why did the young seagull feel dizzy? How would you describe the screams of the gulls in the given
extract? (2m)

Answers: i)d ii) c iii) c iv)He hadn’t eaten anything for a day. Bewilderment

7. If strolling forth, a beast you view,


Whose hide with spots is peppered,
As soon as he has lept on you,
You’ll know it is the Leopard. ’Twill
do no good to roar with pain, He’ll
only lep and lep again.

(How to tell wild animals)


i. Given below are four examples of activities that Jasmeet does. Choose the option that correctly
demonstrates ‘strolling’. (1m)

55
a) Jasmeet runs with a great speed after being chased by a dog.
b) Jasmeet walks in the garden, relaxing while listening to his favourite song.
c) Jasmeet skids sharply on the icy skate rink.
d) Jasmeet rushes to switch off the water pump in the backyard.

ii. Which option lists the statement that is NOT TRUE according to the extract? (1m)

a) The poet asks the reader to hide on seeing the leopard.


b) The poet cautions the reader about a leopard when walking through its territory.
c) The poet informs the reader that a leopard can launch repeated attacks.
d) The poet tells the reader that a leopard attack can result in pain.

iii. The repetition used in “he’ll only lep and lep again” is an example of

a) poetic justice.
b) Satire
c) allusion.
d) poetic licence (1m)

iv. Name the poet. When does one come to know that it is a leopard? (2m)
Answers: i) b ii) a iii) d iv) Carolyn Wells is the name of the poet. According to the poet, one comes to
know that it is a leopard when it leaps or jumps on him.

8. What is the boy now, who has lost his ball,


What, what is he to do? I saw it go
Merrily bouncing, down the street, and then Merrily over- there it is in the
water! (the ball poem)

i. The extract suggests that the poet is

a) an onlooker observing
b) a parent recounting the incident
c) the boy talking about himself
d) imagining the incident (1m)

ii. The poet seems to have indicated the merry bouncing of the ball to

a) create a sense of rhythm in these lines.


b) support the happiness of the experience of playing.
c) contrast with the dejected feeling of the boy.
d) indicate the cheerful mood of the boy. (1m)

iii. Choose the situation that corresponds to the emotion behind the exclamation mark in the poem.(1m)
a) option 1
b) option 2
c) option 3
d) option 4

iv. What is an alliteration? Pick out a line which showcases an example of alliteration from the extract.
Answers: i)aii) a iii) b iv) Alliteration is a literary device that occurs with the same letter or sound atthe
beginning of adjacent or closely connected words. ‘I saw it go’

9. “’Paper has more patience than people’ I thought of this saying on one of those days when I was
feeling a little depressed and was sitting at home with my chin in my hands, bored and listless,
wondering whether to stay in or go out. I finally stayed where I was, brooding: Yes, paper does have
more patience, and since I’m not planning to let anyone else read this stiffbacked notebook grandly
referred to as a ‘diary’, unless I should ever find a real friend, it probably won’t make a bit of
difference.” (Diary of Anne Frank)

i. Pick the option that is ODD out concerning the meaning of ‘patience’. (1m)

a) option (i)
b) Option (ii)
c) Option (iii)
d) Option (iv)

ii. Pick out the emoticon which brings out the meaning of ‘listless’ as used in the extract.(1m)

i) ii) iii) iv)

57
a) a) Option (i)
e) Option (ii)
f) Option (iii)
g) Option (iv)

iii. If the diary were a ‘real friend’, what qualities would Anne expect it to have? Pick the option that lists
these correctly. (1m)

1. optimistic
2. good listener
3. confidant
4. energetic
5. non-judgemental
6. outgoing
7. ambitious
a) 1 & 4
b) 5, 6 & 7
c) 2, 3 & 5
d) 3 & 7
iv. ‘Paper has more patience than people’ – Why did Anne Frank say that? (2m)
Answers:
i) a) option (i) ii) c) option (iii) iii) a) 2, 3 & 5 iv) Anne Frank said that ‘Paper has more patience
than people’ because one can rely on the paper to confide his/her secrets and it listens carefully and
silently.

10. (I am an orphan, roaming the street.


I pattern soft dust with my hushed, bare feet.
The silence is golden, the freedom is sweet.) (Amanda)

i. The tone of the given lines is


a) analytical.
b) despairing.
c) peaceful.
d) nervous(1m)

ii. Read the statements A and B given below, and choose the option that
correctly evaluates these statements.(1m)
Statement A – The figure ‘I’ imagines a less than realistic view of being an
orphan.
Statement B – The figure ‘I’ does not like the speaker.
a) A is true, B is false, according to the extract
b) A is true, B cannot be clearly inferred from the extract.
c) A cannot be clearly inferred from the extract, B is
false.
d) A is true and can be inferred from the poem, B is
true too.
iii. The golden silence is contrasted with the __________. (1m)
a) chaos of the street.
b) constant instructions received.
c) sweetness of freedom.
d) hushed, bare feet.

iv. What is Amanda up to in this stanza? (2m)


Answers: i) (c) ii) (b) iii). (b) iv) Amanda is again taking refuge in her imagination. Here, she wishes
to be an orphan, away from her nagging parents. Amanda wants to roam aimlessly in streets and draw
patterns using just her bare feet.

11. “Our elders are often heard reminiscing nostalgically about those
good old Portuguese days, the Portuguese and their famous loaves of
bread. Those eaters might have vanished but the makers are still
there. We still have amongst us the mixers, the moulders and those
who bake the loaves. Those age-old, time-tested furnaces still exist.
The fire in these furnaces has not yet been extinguished. The thud and
jingle of traditional baker’s bamboo, heralding his arrival in the
morning, can still be heard in some places. ” (glimpses of India)

i. The narrator says that the furnaces were ‘time-tested’ because


e) they had been thoroughly tested each time, before being used.
f) they had proved the test of time and were working well.
g) they had been tested by modern day experts.
h) they had the power to withstand inexperienced usage (1m)

ii. Those eaters might have vanished but the makers are still there.(1m)
Pick the option that expresses the tone of the narrator.
1) elated
2) morose
3) nostalgic
4) hopeful
5) sarcastic
6) critical
7) celebratory
a) 1 and 7
b) 2 and 6
c) 3 and 4
d) 4 and 5

iii. The ‘fire in the furnaces has not yet been extinguished’ implies that
e) the furnaces are still being used to bake the loaves of bread.
f) The fire is in the process of being reviewed as a replaceable method for
heating furnaces.
g) The furnaces are very strong and cannot be shifted for use in other
areas.
h) The fire in the furnaces takes a long time to cease burning, once
59
lighted. (1m)

iv. How do we get to know that the makers of bread still exist? (2m)

Answers: i. b) they had proved the test of time and were working well
ii. c) elated 3. iii) a trip down memory lane iv) The narrator states that the eaters of loaves might have
vanished but the makers are still there. He further says that those age old, time tested furnaces still exist
and the fire in the furnaces had not yet been extinguished.

12. “Day after day she watched the bus, and gradually a tiny wish crept into
her head and grew there: she wanted to ride on that bus, even if it was just
once. This wish became stronger, until it was an overwhelming desire. Valli
would stare wistfully at the people who got on and off the bus when it
stood at the street corner. Their faces would kindle in her longings,
dreams, and hopes. If one of her friends happened to ride the bus and tried
to describe the sights of the town to her. Valli would be jealous to listen
and would shout, in English: “Proud! Proud!” (Madam rides the bus)

i. What does the line ‘gradually a tiny wish crept into her head and grew there’,
mean?(1m)
i) The wish developed in her head over a period of time.
j) The wish was a feeling of wanting to prove her strength.
k) The wish was planted in her head by someone’s suggestion.
l) The wish was small and was overpowered by a sense of doubt.

ii. A part of the extract has been paraphrased. Choose the option that includes the
most appropriate solution to the blanks.(1m)
The desire became so (i) ______________________ that it transformed into an
ii) ____________________________ one. Valli would look (iii)
_________________________ at people who boarded the bus and got off.
a) i) active ii) overt iii) longingly
b) i) vigorous ii) overpowering iii) cheerfully
c) i) staunch ii) overt iii) joyfully
d) i) vigorous ii) overpowering iii) longingly

iii) Pick the option that shows the list of words that collocate with ‘overwhelming’
(e.g. overwhelming desire)(1m)
1. response
2. lies
3. support
4. majority
5. pets
6. places
a) 1, 5 & 6
b) 1, 3 & 4
c) 2 & 5
d) 3, 5 & 6

iv. Why would Valli stare at the people? Why was she jealous? (2m)
Answers: i) a) The wish developed in her head over a period of time. ii) d) i) vigorous ii) overpowering iii)
longingly iii) 1, 3 & 4 iv) Valli would stare at people because she had strong desire to take a ride on
the bus. She had a longing to ride the bus.

13. “Ink trickled down to the bottom of the household,


And little mouse Blink strategically mouseholed.
But up jumped Custard, snorting like an engine,
Clashed his tail like irons in a dungeon,
With a clatter and a clank and a jangling squirm,
He went at the pirate like a robin at a worm” (The Tale of Custard the Dragon)

i. Which option lists the quotes that support the ideas in the extract?(1m)
1) Fear makes strangers of people who would be friends.
2) If you’re brave enough to start, you’re strong enough to finish.
3) Courage doesn’t mean you don’t get afraid. Courage means you don’t let fear
stop you.
4) You get in life what you dare to ask for
5) Fear has a large shadow, but he is strong.
a) 1 and 5
b) 2, 3 and 4
c) 2 and 3
d) 1,3 and 5

ii. What is the poet’s purpose in using the onomatopoeic words given in the extract?(1m)
e) It is to emphasize the might and boldness of Custard.
f) It is to introduce the character Custard to the readers.
g) It is to impress upon the readers that Custard was ready.
h) It is to make Custard bold enough to face the situation.

iii. Pick an option that best fits the usage of the word’ trickled’ as used in the extract.(1m)
e) The water trickled down the tap and filled the trough.
f) Students trickled into the classroom as the teacher entered.
g) Tears trickled down her cheeks as she heard the sad news.
h) His enthusiasm for the task slowly trickled away.

iv. What happened when the pirate appeared? (2m)

Answers: i) c) 2 and 3 ii) a) It is to emphasize on the might and boldness of Custard. Iii) d) …trickled down
to the bottom… iv) The three pets, the cat, the dog, and the mouse, who would boast about their
bravery, ran away and hid themselves. They were the ones who pretended to be fearless. On the other
hand, Custard, the dragon, attacked the pirate bravely. He swallowed the pirate and saved everyone from
danger.

14. Poor Kisa Gotami now went from house to house, and the people pitied
61
her and said, “Here is mustard-seed; take it!” But when she asked, “Did
a son or daughter, a father or mother, die in your family?” they
answered her, “Alas! the living are few, but the dead are many. Do not
remind us of our deepest grief.” And there was no house but some
beloved one had died in it. (the Sermon at Benares)

i. The community’s response to Kisa in the above extract was somewhat


different from before. Why do you think that was the case?(1m)
a) They had learnt from Buddha’s sermons.
b) They were able to help Kisa in some way this time.
c) They understood parental grief.
d) They liked Kisa and enjoyed talking to her.

ii. Which of the following options represents the correct understanding of the
word “poor” in the phrase “Poor Kisa Gotami”?(1m)
a) in need of money
b) weak
c) unfortunate
d) inferior

iii. Pick the option that explains — ‘…the living few, but the dead many.’(1m)
a) It shows the high death rate and low birth rate in the
city of Benares.
b) It highlights the holy status of Benares where many Hindus go to die.
c) It throws light on the numerous loved ones the villagers had lost over time.
d) It reflects that many children who had died in the village
for various reasons.

iv. What did the people reply Kisa Gotami when she asked about the death of a family member? (2m)

Answers: i) b ii) c iii. c iv. The people replied Kisa Gotami that there was no house but some beloved
one had died in it and she should not remind them of their deepest grief.

14. “I heard an old religious man


But yesternight declare
That he had found a text to prove
That only God, my dear,
Could love you for yourself alone
And not your yellow hair.” (Anne Gregory)

i. ‘yesternight’ is a word made of combination of two words to form a new word.


Hence, it is a compound word.(1m)
Choose the words that DO NOT fit the description.

a. 3, 4
b. 2, 6
c. 3, 5
d. 1, 4
ii. The above stanza strongly defies the idea of ___________ .(1m)
a. internal qualities
b. external beauty
c. spiritual self
d. divinity

iii. Choose the option that displays the likely image of the man mentioned in the
extract. (1m)

a) option 1
b) option 2
c) option 3
d) option 4

iv. Who had found a ‘text’? What does the text prove? (2m)

Answers: i) b ii. B iii) c iv) An old religious man had found a text.
(b) The text proves that only God is capable of looking beyond external beauty, into the soul of a person.

15. LOMOV: Never mind about my people! The Lomovs have all been
honourable people, and not one has ever been tried for embezzlement, like
your grandfather!
CHUBUKOV: You Lomovs have had lunacy in your family, all of you!
NATALYA: All, all, all!
CHUBUKOV: Your grandfather was a drunkard, and your younger aunt,
Nastasya Mihailovna, ran away with an architect, and so on...
LOMOV: And your mother was hump-backed. [Clutches at his heart]
Something pulling in my side... My head.... Help! Water!
CHUBUKOV: Your father was a guzzling gambler! (The Proposal)

i. Choose the option that correctly identifies the tone of the characters in the
given extract.(1m)
1) antagonism
2) humour
3) contempt
4) irony
63
a) (1) and (2)
b) (2) and (4)
c) (1) and (3)
d) (3) and (4)

ii. Imagine you found the playwright’s notes for each scene in the play and
noticed that some of the words were missing.
Choose the option that fills the missing words most appropriately.
A conversation that starts pleasantly quickly turns into a (i) ________
argument. With (ii) _________ of Oxen Meadows at the heart of the matter, Lomov and Natalya quarrel
and are later joined by Chubukov. Thus, begins
a (iii) ____ of insults, accusations and name-calling. All (iv) _______
disappears. Eventually, Lomov leaves clutching at his heart, his foot numb.
a) (i) petty ; (ii) history ; (iii) series; (iv) politeness
b) (i) vicious ; (ii) ownership ; (iii) circus ; (iv) civility
c) (i) curious ; (ii) land ; (iii) outpouring ; (iv) laughter
d) (i) ugly ; (ii) neighbourhood ; (iii) barrage; (iv) goodness

iii. Which of the following options comes closest to the meaning of ‘tried’ as
used in the extract?
a) She mastered the game through a process of trial and error.
b) He followed the trial closely and was seen in court every day.
c) This had been a tried and tested formula for the organisation.
d) They tried with all their might to repeat their earlier successes

iv) What is the intention of the playwright in the given extract? Why does Chubukov considers Lomovs
proposal if they are not honorable people?

Answers: i) c ii) b iii) b iv) to emphasize that family history is important in a marriage proposal. Because
Lomov’s status in society supersedes everything.

16. The fog comes


on little cat feet.
It sits looking
over harbour and city
on silent haunches
and then moves on. (Fog)

i. Choose the option with qualities of the cat that Carl Sandburg applies to the
fog? (1m)
1) Cats are independent animals, they don’t follow rules, they slip and slide in and out of our
lives as they please.
2) Cats are distrustful of strangers and can be jealous and moody.
3) Cats are stealthy, moving in slow motion at times and they appear to be moving in a
mysterious fashion.
4) Cats often communicate with a combination of a distinctive sound and
body language.
5) Cats like to move on at their own pace and before you know it, they've disappeared.
a) Only 1
b) 2, 3 and 4
c) 1,3 and 5
d) Only 4

ii. Pick the option that includes an image of the cat on its haunches. (1m)

a. Option (1)
b. Option (2)
c. Option (3)
d. Option (4)

iii. Choose the option that DOES NOT list the movement of the fog. (1m)

a. Option (1)
b. Option (2)
c. Option (3)
d. Option (4)

iv. What does the poet mean by little cat feet? (2m)

Answers: i) c ii) b iii) c iv) Little cat feet here represents the silent and careful steps of a cat. The way fog
comes, resembles the steps of a cat.

65
17. For the first twenty-four hours Mijbil was neither hostile nor friendly;
he was simply aloof and indifferent, choosing to sleep on the floor as
far from my bed as possible. The second night Mijbil came on to my
bed in the small hours and remained asleep in the crook of my knees
until the servant brought tea in the morning, and during the day he
began to lose his apathy and take a keen, much too keen, interest in
his surroundings. I made a body-belt for him and took him on a lead to
the bathroom, where for half an hour he went wild with joy in the
water, plunging and rolling in it, shooting up and down the length of
the bathtub underwater, and making enough slosh and splash for a
hippo. (Mijbil, the otter)
i. Mijbil was neither hostile nor friendly; he was simply aloof and indifferent…” (1m)

From the actions of the given set of friends, choose the person who’s
behaviour is likely to resemble Mijbil’s.
e) Akanksha rudely tells her best friend to stop calling.
f) Piyush was amicable since the day he walked into his new classroom.
g) Rishabh became nasty as soon as he started earning money.
h) Urvashi was distant for a while after losing her grandmother.

ii. Select the option that correctly puts the following events in correct order. (1m)

1. Maxwell planned to take Mijbil to the bathroom.


2. Mijbil slept at a significant distance from Maxwell’s bed.
3. Mijbil immensely enjoyed playing in water.
4. Mijbil slept in much closer vicinity of the author as opposed to the
previous night.
a) 1, 3, 4, 2
b) 3, 1, 2, 4
c) 2, 4, 1, 3
d) 4, 3, 1, 2

iii. Select the option that best describes the final action of the otter, based on
the given passage. (1m)

a) Option (1)
b) Option (2)
c) Option (3)
d) Option (4)

iv. What was the first characteristic of the otter discovered by the narrator? (2m)

Answers: i) d ii) c iii) b iv) Mij spent most of his time in play. He spent hours shuffling the rubber ball
round the room like a four-footed soccer player. His real play was when he used to juggle small objects
between his paws. Marbles were his favourite toy for his pastime.

18. I sit inside, doors open to the veranda


writing long letters
in which I scarcely mention the departure
of the forest from the house.
The night is fresh, the whole moon shines
in a sky still open… (The Trees)

i) The poet talks about the house as


a) a safe sanctuary from predators.
b) a place of confinement.
c) a representation of family.
d) a concrete alternative to a natural on (1m)

ii) The extract uses ‘open’ twice, suggesting different meanings. Pick the option
that lists the correct meanings, respectively. (1m)

a) entry point-vastness
b) sharing a common space-great view
c) allowing access and vision-expanse of space
d) advantage-obstacles ahead

iii) Choose the option that correctly depicts the imagery in the last two lines of the
extract (1m)

a) Option (1)
b) Option (2)
c) Option (3)
d) Option (4)

iv. What is the poet doing? Where are the trees in the poem? (2m)
67
Answers: i) b ii) c iii) c iv) She was writing long letters. The trees are inside the house in the poem.

19. The entire staff was roused and maids rushed in and out bringing his day
bed, his night bed, favourite cushions, toys and rubber rings, breakfast bowl,
lunch bowl, supper bowl. Realising that my car would never hold all the
stuff, I started to drive away. As I moved off, Mrs Pumphrey, with a
despairing cry, threw an armful of the little coats through the window. I
looked in the mirror before I turned the corner of the drive; everybody was in
tears. Out on the road, I glanced down at the pathetic little animal gasping
on the seat by my side. I patted the head and Tricki made a brave effort to
wag his tail. “Poor old lad,” I said. “You haven’t a kick in you but I think I
know a cure for you.” (The Triumph of Surgery)

i. What might the atmosphere of the household in the above extract signify? (1m)
a) Mrs Pumphrey’s status in society reflected in Tricki’s lifestyle.
b) The staff’s love for Tricki, which matched that of Mrs. Pumphrey
c) The grand life of comforts and luxuries that Tricki enjoyed.
d) Mrs. Pumphrey’s indulgence and anxiety acted upon by the staff.

ii) Given below are some well-known quotes shared by the staff to console
Mrs. Pumphrey, after Tricki’s departure. (1m)
Choose the option that correctly identifies the quote that IS NOT appropriate
to the consolation offered.

a) Option (i)
b) Option (ii)
c) Option (iii)
d) Option (iv)

iii. The narrator describes Tricki as a “pathetic little animal”. The use of the
word ‘pathetic’ indicates that the narrator (1m)
a) was very fond of Tricki.
b) thought Tricki was contemptible.
c) pitied Tricki’s condition.
d) believed Tricki’s health was deteriorating.

iv. Why was the entire staff of Mrs. Pumphery at work while Tricki was going to hospital? (2m)

Answers: i) d ii) d iii) c iv) The entire staff of Mrs Pumphrey was at work while Tricki was going to the
hospital, to supply all necessary items of Tricki to the hospital.
20. Anil was watching a wrestling match when I approached him. He was about
25-- a tall, lean fellow – and he looked easy-going, kind and simple enough
for my purpose. I hadn’t had much luck of late and thought I might be able
to get into the young man’s confidence.
‘You look a bit of a wrestler yourself,” I said. A little flattery helps in making
friends. (The Thief’s story)

i) According to the extract, the young boy was watching the wrestling match
because he
a) had been invited there by the wrestlers.
b) was supposed to meet someone there.
c) was looking for simple people to dupe.
d) loved wrestling and followed it very closely. (1m)

ii) ‘I hadn’t had much luck of late’ means that the boy hadn’t (1m)
a) ever conned people successfully.
b) been successful in duping people lately.
c) understood the consequences of thievery till date.
d) considered the role of fate in deceiving others.

iii) Anil looked easy-going, kind and simple to the narrator. (1m)
Which of the given characteristics would NOT fit in with this description?
a) compassionate
b) suave
c) uncomplicated
d) carefree

iv. Who was Anil? (2m)

Answers: i) c ii) b iii) b iv) Anil was a kind , simple and easy-going young man. He was tall and lean
stature and interested in sports like Wrestling .

21. Max was slender, a little less than tall, with features that suggested slightly the
crafty, pointed countenance of a fox. There was about him — aside from the
gun — nothing especially menacing. “The report,” he murmured. “The report
that is being brought to you tonight concerning some new missiles. I thought I
would take it from you. It will be safer in my hands than in yours.” Ausable
moved to an armchair and sat down heavily. “I’m going to raise the devil with
the management this time, and you can bet on it,” he said grimly. “This is the
second time in a month that somebody has got into my room through that
nuisance of a balcony!” (Midnight Visitor)

i. Owning a gun gave a menacing impression to Max’s character because (1m)


a) a gun is only used by police officials.
b) he was carrying it without any official license.
c) a gun has a threatening implication.

69
d) the gun was particularly huge

ii. Choose the option that displays the correct analogy. (1m)
a) crafty : fox :: menacing : gun
b) report : missile :: management : devil
c) crafty : fox :: management : balcony
d) armchair : grim :: balcony : nuisance

iii. Choose the part of the house through which Ausable thought Maxwell had
entered his room. (1m)

a) Option 1
b) Option 2
c) Option 3
d) Option 4

iv. Which word replaces ‘sat down heavily’ in the sentence—'Ausable moved to
an armchair and sat down heavily’? Why did Max come to Ausable’s room? (2m)

Answers: i) c ii) a iii) b iv) Flumped. He had come to Ausable’s room demanding a secret report from
him.

22. How foolish people are when they own valuable things, Horace
thought. A magazine article had described this house, giving a plan of
all the rooms and a picture of this room. The writer had even mentioned
that the painting hid a safe! But Horace found that the flowers were
hindering him in his work. He buried his face in his handkerchief. Then
he heard a voice say from the doorway, “What is it? A cold or hay
fever?” Before he could think, Horace said, “Hay fever,” and found
himself sneezing again. The voice went on, “You can cure it with a
special treatment, you know, if you find out just what plant gives you
the disease. I think you’d better see a doctor, if you’re serious about
your work. I heard you from the top of the house just now.”

i) The line from the extract that can be considered an example of


sarcasm is
a) How foolish people are when they own valuable things.
b) Horace found that the flowers were hindering him in his work.
c) You can cure it with a special treatment.
d) I think you’d better see a doctor, if you’re serious about your work (1m)

ii. Which magazine could have published the article mentioned in the
given extract? (1m)
a) Paws & Claws
b) Outdoors & Landscaping
c) Interiors
d) The Investor

iii. Choose the image that correctly describes the location of the safe,
based on the given extract. (1m)

a) Option (1)
b) Option (2)
c) Option (3)
d) Option (4)

iv. Horace sneezed. Why did he do it? (2m)

Answers: i) d ii) c iii. c iv. Horace did not sneeze deliberately, rather he suffered from an attack of hay
fever. There was a big bowl of flowers kept in the drawing room that caused him to sneeze.

23. “I didn’t get any real results,” he said. “But I went ahead and showed that I had
tried the experiment. This time I won.” The next year his science fair project
was testing the theory that viceroy butterflies copy monarchs. The theory was
that viceroys look like monarchs because monarchs don’t taste good to birds.
Viceroys, on the other hand, do taste good to birds. So, the more they look
like monarchs, the less likely they are to become a bird’s dinner. Ebright’s
project was to see whether, in fact, birds would eat monarchs. He found that a
starling would not eat ordinary bird food. It would eat all the monarchs it could
get. (Making of the Scientist)

71
i. Choose the option listing Ebright’s qualities as depicted by the above extract. (1m)
1. persevering
2. visionary
3. determined
4. liberal
5. conceited
a) 1, 2
b) 3, 5
c) 1, 3
d) 4, 5

ii. According to the dictionary, ‘fair’ as a noun, shows the following meanings.
Choose the option that lists the meaning similar to the usage to that in the
extract. (1m)
a) A gathering of stalls and amusements for public entertainment.
b) A competitive exhibition showcasing products or ideas.
c) A periodic gathering for the sale of goods.
d) An annual exhibition of livestock, agricultural products, etc., held by a
town, county, or state.

iii. Choose the option that is true for the two statements given about the
information in the extract. (1m)
Statement 1- Starling feeds on viceroys.
Statement 2- Starling does not eat seeds and insects.
a) Both statements are clearly mentioned in the extract.
b) Statement 1 cannot be clearly inferred from the text and statement 2 is
true.
c) Statement 1 is false and statement 2 cannot be inferred from the
extract.
d) Both statements need to be inferred from the given extract.

iv. What was Ebright’s project? (2m)

Answers: i) c ii. b iii. c iv. Ebright's project was to see whether birds would eat monarchs. He found
that a starling would not eat ordinary bird food. It would eat all the monarchs it could get. (Ebright said
later research by other people showed that viceroys probably do copy the monarch.)

24. She suffered incessantly, feeling herself born for all delicacies and luxuries.
She suffered from the poverty of her apartment, the shabby walls and the
worn chairs. All these things tortured and angered her.
When she seated herself for dinner opposite her husband who uncovered the
tureen with a delighted air, saying, “Oh! the good potpie! I know nothing
better than that...,” she would think of elegant dinners of shining silver; she
thought of the exquisite food served in marvellous dishes. She had neither
frocks nor jewels, nothing. And she loved only those things.
She had a rich friend, a schoolmate at the convent, who she did not like to
visit- she suffered so much when she returned. She wept for whole days from
despair and disappointment.” (Necklace)
i. Choose the option that list the set of statements that are NOT TRUE according to
the given extract. (1m)
1. Matilda was very pleased with her life.
2. Matilda envied her friend for being well-off.
3. M Loisel didn’t appreciate what Matilda cooked.
4. Matilda despised the fact that she lived a life of poverty.
5. Matilda never felt troubled, though she desired a luxurious life.
6. Matilda thought of grand dinners and silverware sitting at the dinner table.
7. Matilda felt depressed after visiting her friend.
a) 1, 3, 6
b) 3, 5, 7
c) 1, 3, 5
d) 2, 4, 7

ii. Which word does ‘delicacies’ NOT correspond to? (1m)


a) etherealness
b) elegance
c) exquisiteness
d) robustness

iii. Choose the characteristic displayed by M Loisel in the extract. (1m)


a) conceited
b) contended
c) appeased
d) subdued

iv. What did Mdem Loisel suffer from? (2m)

Answers: i) c ii) d iii) b iv) She suffered incessantly, feeling herself born for all delicacies and luxuries.
She suffered from the poverty of her apartment, the shabby walls and the worn chairs. All these things
tortured and angered her.

25. “Ramlal stood rooted to the ground, his head bowed low with the weight of grief and shame.
The flames of the sacred fire slowly died down. Everyone was gone. Ramlal turned to
Bholi and said, “But what about you, no one will ever marry you now. What shall we do with you?”
And Sulekha said in a voice that was calm and steady. “Don’t you worry, Pitaji! In your old age I will
serve you and Mother and I will teach in the same school where I learnt so much. Isn’t that right,
Ma’am?”
The teacher had all along stood in a corner, watching the drama. “Yes, Bholi, of course,” she replied.
And in her smiling eyes was the light of a deep satisfaction that an artist feels when contemplating the
completion of her masterpiece.” (Bholi)

i. Ramlal stood rooted to the ground because he


a) was moved by what he heard.
b) was influenced by Bholi’s words
c) was in a state of shock.
73
d) was in an immovable position. (1m)

ii Bholi had refused to get married as


a) her father couldn’t afford the dowry that was demanded
b) the bridegroom had been greedy and was disrespectful
c) the bridegroom had insulted her father
d) her father was getting her married to a man older to her (1m)

iii Pick the sentence that brings out the meaning of ‘contemplating’ as used in the extract. (1m)
d) Contemplating sharing my belongings with someone is definitely tough.
k) She took some time to respond as she was contemplating what to say.
l) I was contemplating my reflection in the mirror and was speechless.
m) She was contemplating though the pages of the document that was with her.

iv. Don’t you worry, Pitaji! In your old age I will serve you and mother.” Through the statement the
narrator wants to highlight moral values Bholi was imbued with. Based on the reading of the lesson, what
made Bholi aware of her rights and how did she use them ? (2m)

Answers: i) c ii) b iii) b iv)Self -confidence, love and affection for parents, self-respect, determination,
the teacher made her aware of her rights, rejected the proposal of the greedy man, decision making
power.

26. THINK-TANK: Mirror, mirror, in my hand. Who is the most fantastically


intellectually gifted being in the land?
OFFSTAGE VOICE: (after a pause) You, sir.
THINK -TANK: (smacking mirror) Quicker. Answer quicker next time. I hate a
slow mirror. (He admires himself in the mirror.) Ah, there I am. Are we
Martians not a handsome race? So much more attractive than those ugly
Earthlings with their tiny heads. Noodle, you keep on exercising your mind,
and someday you’ll have a balloon brain just like mine.
NOODLE: Oh, I hope so, Mighty Think-Tank. I hope so.
THINK -TANK: Now, contact the space probe. I want to invade that primitive
ball of mud called Earth before lunch.
NOODLE: It shall be done, sir. (He adjusts levers on switchboard. Electronic
buzzes and beeps are heard as the curtains open.) (The Book that Saved the Earth)

i.Choose the option that lists the most appropriate meaning of think-tank from
those given below. (1m)
a) A tank that helps you think deeper than an average person, if you use
its contents regularly.
b) Non-stop flow of ideas from a fixed source about one particular issue.
c) A pool of ideas and solutions to various problems via an individual,
group or organisation.
d) A method by way of which people think collectively to identify stray
thoughts among them.

ii) ‘Mirror, mirror, in my hand. Who is the most fantastically intellectually gifted
being in the land?”(1m)
Choose the option that lists the genre having the original reference to the
above lines.

a) Biography
b) Science-fiction
c) Mystery
d) Fairy tale

iii. Choose the option that correctly categorizes the phrase “(after a pause)”
based on the dialogue given above. (1m)
a) aside
b) stage direction
c) narration
d) setting

iv. Give an example of irony from the passage. What does Noodle tell Think-Tank about the books ? (2m)

Answers: i) c ii. D iii. b iv. “Noodle, you keep on exercising your mind, and someday you’ll have a
balloon brain just like mine’’. Initially Think-Tank consider books as a crude refreshment stand.

LONG ANSWER QUESTIONS:


Answer the following in about 100- 120 words.
FIRST FLIGHT
(a) 'As ripe fruits are in danger of falling early, so mortals when born are always in danger of death'.
With this statement of the Buddha, find out the moral values that Kisa Gotami learnt after the death of her
child.

(b) 'No one is born hating another person because of the colour of his skin, or his background or his
religion'. Do you agree? Elaborate on the basis of the chapter "Nelson Mandela-Long walk to freedom".

(c) Motivation plays an important role in taking risks in life and in succeeding. Do you agree? Discuss
with reference to the pilot of the old Dakota and the young seagull.

(d) Valli was an extra-ordinary girl who had self confidence and courage to realise her ambition by
planning and drawing on herspirit of adventure. Discuss in reference to the chapter 'Madam Rides the Bus'.

(e) You have been asked to present an evaluation of the approaches of the mothers of both, the baby seagull
and Amanda, towards helping their children. Write this presentation draft including your insights, in about
120 words, comparing the approaches of both parents.
You may begin this way:
One acknowledges that both parents, Amanda’s mother and the baby seagull’s mother both....however, ...
(Reference -Amanda! & His First Flight)

(f) You have been chosen to address a student gathering from the neighbourhood schools, to speak on the
resilience of human spirit required to transcend discrimination. Prepare the speech draft in not more than
75
120 words, with reference to the commonality of themes in Nelson Mandela: Long Walk to Freedom and
The Trees by Adrienne Rich.
You may begin this way:
Good morning, everyone. Today, I'd like to discuss two pieces of literature that offer a powerful insight
into the resilience of the human spirit required to transcend discrimination.
You may end this way.
To conclude, I’d like to say that ...
Thank you

g) “Difficulties do come in our lives, but they are not there to stay forever.” Comment on this statement
with reference to the poem ‘Fog’ by Carl Sandburg.

h) One is known by one’s roots. The people of Coorg are also recognized for their values and traits in spite
of being thousands of miles away from their places of
origin-Greece and Arabia. Describe their values and traits.(Glimpses of India)

i) Explain the phrase “the forest that was empty all these days”. After reading the poem “The Trees”, for
whom do you think are the forests needed? Imagine you are a tree in a forest, what values would you like
the humans to learn from the tree?

j) If the Buddha were to summarise the life lessons of ‘the Ball poem’ what would that sermon be? Think
and create this address for people of your age.

k) “If you don’t help me, my family and I will go hungry this year.” Lencho had faith in God but he didn’t
manage to solve the problem by himself. Did he lack the courage to resolve his matter himself? What
values did he lack ? Explain.(A Letter to God)

l) Being impressed by Custard’s feat, the young seagull’s sister wrote a letter to Custard seeking advice on
addressing the situation with her young brother. Write Custard’s response to this letter. Base the advice
from his experiences.
You may begin like this:
Dear Ms Seagull,
Thank you for seeking me out. I am humbled. Based on my limited experiences I feel...
(The Tale of Custard the Dragon)

m) “Water must be kept on the move and made to do things; when static it is wasted and provoking.” Do
you think this statement was true in case of Mijbil, the otter ?(Mijbil the Otter)

n) Imagine Anne Gregory grows up to be a motivational speaker. As Anne Gregory, write a short speech on
the importance of loving oneself.
You may begin this way:
Dear friends
The importance of believing in oneself can never be overstated. It becomes most important, therefore, to
love oneself. I learnt this, as a young teen, the day a
poet pointed out that (continue) .....(For Anne Gregory)
FOOTPRINTS WITHOUT FEET
a) Destiny had been cruel to Bholi yet she made a place for herself in a conservative society. Discuss.

b) Contentment in one's life is very important to lead a peaceful life. We should be happy with what
we have and should not crave for what we don't have. Matilda suffered in life because she was not content
in her life. Her desires led to her disaster. What do you learn from her mistake in life?

c) Trust and compassion can reform a person. Justify this statement in the light of the lesson 'The Thief's
Story'.

d) Imagine that M. Loisel, from The Necklace by Guy de Maupassant, writes a diary entry, exploring the
theme of class and social status, and the nature of social mobility, in the context of his own experience.
Write this diary entry, as M. Loisel, in about 120 words.

e) A character arc is the transformation or development of a character throughout a story and refers to the
changes a character undergoes as a result of their experiences, challenges, and interactions with other
characters.
In the light of the above information, trace the character arc of the thief in Ruskin Bond’s The Thief’s
Story, in about 120 words.

f) Albert Einstein said, ‘The important thing is to never stop questioning.’ Richard was a genius who
proved this quote true. Justify.(The Making of a Scientist)

g) Give a brief character sketch of Mrs. Pumphrey. What values are reflected through her character ?
(A Triumph of Surgery)

h) “Once bitten twice shy.” Do you think Horace would ever be able to believe any one ever in his life?
Comment.(A Question of Trust)

i) Give a brief character sketch of Fowler? What are the values reflected in his character? (The Midnight
Visitor)

j) Give a character-sketch of Hari Singh. (The Thief’s Story)

k) Who do you think is the real culprit in the story ‘A Question of Trust’? Explain.

l) Readers believe that Griffin was a man without a conscience. You have a class discussion on why our
conscience plays a vital role in shaping our character.
Present a summary of your views on this. (Footprints Without Feet)

77
m) Attempt character sketch of Great and Mighty Think- Tank.
***************
ANSWERS -First Flight

a. It is a fact that the human life is mortal. It is brief and has pain. Death is a reality which has
to be accepted by one and all. As ripe fruits are early in danger of falling, so mortals when born
are always in danger of death. Whether its’s a young or an adult, fools or wise, all are subject
to death.
After the death of her child, Kisa Gotami learnt that death is inevitable and cannot
be reversed. Whether somebody accepts it willingly or unwillingly, it’s a fact which needs
to be accepted. When she roamed around in the village from house to house, she found
that there was no house where the members were immortal. Those who have taken birth are
bound to die one or the other day. Just like ripe fruits are likely to fall apart one day, similarly
humans also go far away after dying. Fruit once fallen from a tree cannot be hung back
similarly the human life once gone cannot be brought back.
Thus, Kisa Gotami concluded after the Budhha made her learn indirectly that death
is inevitable for all the living creatures on this earth.
b. According to Nelson Mandela, no one is born hating another person because of the colour of his skin, or
his background, or his religion. People learn to hate, and if they can learn to hate, they can be taught to
love, for love comes more naturally to the human heart than its opposite. Even in the grimmest times in
prison, when Nelson Mandela’s comrades and he were pushed to their limits, he could see a glimmer of
humanity in one of the guards, perhaps just for a second, but it was enough to reassure him and keep him
going. Man’s goodness is a flame that can be hidden but never extinguished. I agree with him. Children
become what they learn from their environment. Love is stronger than hate.

c. Motivation plays a very important role in taking risks in life and succeeding. Motivation
is what urges people to take actions or risks to achieve their goals. Without motivation, it
is not possible to reach the aims in life and be successful. The pilot of old Dakota is motivated
to meet his family and have dinner with them. So, when he sees dark clouds ahead, he does
not stop and risks his as well as the life of passengers to reach hid destination.
The young seagull is afraid of flying. The seagull would go hungry because no one from
his family gave him food. He saw others flying and when once risked to jump to take food
from his mother, realised that he had to fly to live. Hunger is what motivated the young
seagull to fly.

d. Valli is an extraordinary girl who is self-confident and has an adventurous spirit that
she uses to release her ambitions of seeking different adventures. As a young girl, she is
curious and wants to learn a lot from others. She is a keen observer and listens to conversations to gain
knowledge from them. She also asks a lot of questions and sometimes about different
places and journeying to them. She even befriends a conductor to get knowledge to fulfil
her hunger for adventure. It is through all her planning and ability to know and learn that she
is able to achieve her dreams of adventures.

e. The baby seagull’s mother:

The baby seagull’s mother was a parent who wanted to better her child's life by pushing him out of his
comfort zone and encouraging him to learn new skills. She believed that her son is capable of flying, even
though he was afraid, and tried to motivate him by showing him how much fun it could be. She also offered
him scraps of fish to reward him for his efforts, which is truly a kind of positive reinforcement strategy.
However, the mother's approach also had some drawbacks. By constantly pushing the baby seagull to fly,
she was putting him in danger and risking his physical well-being. She also didn't seem to acknowledge his
fear and anxiety, which could be a source of stress for the baby seagull.

Amanda’s mother:

Amanda's mother is depicted as a strict and critical figure who is presented as always correcting Amanda's
behavior, from her posture and habits to her academic performance and personal hygiene.

While her intentions may be good, her methods are shown to be ineffective, and in some cases, harmful. By
constantly nit-picking Amanda's behavior, her mother creates an atmosphere of tension and anxiety, which
only serves to make Amanda feel worse about herself. This also causes Amanda to feel like she can never
measure up, leading her to seek solace in imaginary worlds and fantasies.

The methods of Amanda's mother do not come across as conducive to achieving this goal. Rather than
focusing on Amanda's strengths and encouraging her to develop them, she highlights her weaknesses and
shortcomings. Amanda is thus, left feeling undervalued, which could have a negative impact on her self-
esteem and mental health.

Comparison:

In comparison to Amanda's mother, the seagull's mother seems to be more attuned to her child's needs and
abilities. She recognizes that the baby seagull is capable of flying and wants to help him achieve his full
potential. However, she may not be taking into account his fears and anxieties, which can be just as
important to his well-being as his ability to fly. In contrast, Amanda's mother seems to be too focused on
correcting her daughter's flaws and may not be paying enough attention to her strengths and abilities.

Even though both mothers have good intentions and want to better their children's lives, their methods, in
part, may be misguided or ineffective. The seagull's mother could benefit from being more sensitive to her
child's emotions, while Amanda's mother could benefit from focusing on her daughter's strengths and
building her self-esteem.

Ultimately, the best approach for both parents would be the one that would balance positive reinforcement
with sensitivity to the child's needs and limitations.

f. Good morning, everyone. Today, I'd like to discuss two pieces of literature that offer a powerful insight
into the resilience of the human spirit required to transcend discrimination. Both works share some
common themes.

(Theme 1: Transcending Discrimination)

Both Mandela's excerpt and Rich's poem address the issue of discrimination. Mandela speaks of how his
own experiences of discrimination made him more determined to fight against it. He emphasizes the need
to move beyond the divides created by race, gender, and class. Similarly, Rich's poem acknowledges the
discrimination faced by trees, which are often overlooked and undervalued. She argues that these trees
deserve to be recognized and appreciated, just as all living beings should be. [The metaphorical perspective
for The Trees - the emergence of women against discrimination. Rich compares the growth of trees to the
growth of women who were once suppressed and marginalized by society. Just as trees grow from the earth
and reach for the sky, women too are rooted in their past but strive towards a better future, breaking free
from the chains of oppression and discrimination.]
79
(Theme 2: The Efforts Involved in Achieving Equality)

Both pieces of literature also highlight the hard work and sacrifices required to achieve equality. Mandela
describes his long and difficult journey toward freedom, including his time spent in prison. He emphasizes
that true freedom is not just about breaking physical chains but also breaking mental ones. Rich's poem
speaks of the efforts required to protect trees and nature, highlighting the importance of activism and
advocacy. [The metaphorical perspective for The Trees - Despite being cut down and uprooted by the
patriarchy, women continue to grow and flourish, resisting oppression and striving towards equality. The
trees symbolize the tenacity and determination of women to rise above their circumstances and claim their
rightful place in society.]

(Theme 3: Resilience of the Human Spirit)

Despite the challenges faced, both Mandela's excerpt and Rich's poem showcase the resilience of the
human spirit. Mandela's perseverance and unwavering determination to fight against discrimination, even
in the face of great adversity, inspire us to do the same. Rich's poem also speaks to the resilience of nature,
which continues to thrive even in the face of human neglect and disregard. [The metaphorical perspective
for The Trees - The poem celebrates the strength and tenacity of women and highlights the importance of
perseverance in the face of adversity. Through the symbol of the trees, Rich suggests that Content 3
Expression 2 Accuracy 1 13 of 14 just as nature can regenerate and thrive, so too can women emerge
stronger and more resilient from discrimination and oppression.]

Conclusion:

To conclude, I’d like to say that the common themes of transcending discrimination and the efforts
involved in achieving equality are prevalent in both these pieces of literature and remind us of the strength
of the human spirit and the importance of standing up for what we believe in, even when faced with
obstacles. By acknowledging and valuing all forms of life and working towards a more just and equal
world, we can continue to build a better future for ourselves and for future generations.

Thank you.

g. When people face difficulties in their lives, at the end, these problems tend to leave them hopeless and
shattered. It takes a lot of courage for anyone to overcome their problems and to even try to solve it. But if
we take a lesson from this poem ‘Fog’ and compare our difficulties also come and go. They are not a
permanent resident in our lives and hence, don’t stay forever.

There is no need for us to become hopeless and lose courage when are faced with problems. We should
rather try to think of those difficulties or problems as a fog, meaning that think of it as something that has
not come to stay for long, but something will always leave, like every other bad period of our lives.

h. It is true that one is known by one’s roots. So, in the context, the fiercely independent people of Coorg
are possibly of Greek or Arabic origin. The Coorgis have their roots in their places of origin, that is, Greece
and Arabia. So, their blood or culture is apparent in their martial traditions.

They are more than willing to recount numerous tales of valour related to their sons and fathers. The Coorg
Regiment is one of the most decorated in the Indian Army. Although Coorgis are thousands of miles away
from their places of origin – Greece and Arabia, they are virtually recognized for their values and traits.
They have tradition of hospitality. They are brave, fierce, independent and daredevil. This quality makes
them unique.
i. In the poem, ‘The Trees’ poet Adrienne Rich subtly drives home the message about the importance of
trees. Without trees, the birds would not have a place to sit, insects will have no place to hide and the sun
would not bury its feet in shadow. As saplings, we enjoy the beauty of plants as they adorn the
surroundings. But slowly, the tree spreads its roots, its branches and leaves, and seems to yearn to go
outside where it can live and grow without any restrictions. No more does the tree look attractive indoors.
The trees are however welcomed into nature by strong winds and the moon. The poet hereby emphasises
that trees need to be kept alive, but should not be ‘imprisoned’ inside the house as they look more beautiful,
and tend to thrive outdoors that is where trees belong.

j. Losses are integral part of life. We must look at them in totality. First of all, we must always keep in
mind that life’s each and everything is God’s gift for which we must be thankful. The things which God has
given can be taken back anything. We must not be attached with these gifts. Once these attached gifts are
taken away, we become sad we feel the lost thing is irreplaceable with any other things.

The lost ball stands for the general losses a human being suffers as he grows old. The losses may be the
loss of a personal possession or the death of a dear one or separation from a beloved one. As long as there
is life, there will be many types of losses; what each one has to learn is bearing those losses.

k. Lencho was a hardworking farmer. He was completely dependent upon the crops of his fields. His
annual crop was completely destroyed due to heavy rains and hailstorm. In the entire village, there was no
one to help him during the times of financial crisis. He had the firm belief in God and hence wrote a letter
requesting Him to send 100 pesos so that he could sow his fields again till the next crop. This act did not
prove that he lacked the courage to resolve his problem but shows that he did not have any option left for
himself He had a firm belief in God. He was an ox of a man but lacked money to stand again.

l. Dear Ms. Seagull.

Thank you for seeking me out. I’m humbled. Based on my limited experiences, I feel that what a human
actually possess in him can only be proven at the times of utmost difficulty. In my case, not only Belinda
and the other pets took me as a coward, but also, I was unaware of my own courage. It was only when I
saw Belinda crying for help on the sight of a pirate breaking in; I knew that I had to do it at any cost. So, I
without thinking, I charged at the pirate with all my strength. Similarly, I believe that when your younger
brother will face such a situation where he would be left with no choice but to fly, he would definitely take
the plunge. He was too unaware of the courage he had in himself like me. But he would soon prove himself
to everyone.

M. Yes, I think this statement is true in the case of Mijbil, the otter. Maxwell took Mijbil on a lead to
the bathroom and Mij just went wild with joy while in water. Mij was plunging and rolling in water. He
was even shooting up and down the length of the bathtub underwater and was making too much slosh
and splash.
This, Maxwell learnt, is a characteristic of otters. According to them, every drop of water must be
extended and spread about the place. It was further proved when two days later, Mij escaped from
Maxwell’s bedroom as Maxwell entered it. Maxwell followed Mijbil who had gone to the bathroom and
was fumbling at the chromium taps with his paws. Maxwell watched with surprise when Mij, turned the
tap, which ultimately achieved full flow. This shows that when an otter is in the water, water must be
kept on the move and made to do things because, according to the otter, otherwise it is wasted.

n. Dear friends
The importance of believing in oneself can never be overstated. It becomes most important therefore to
81
love oneself. I learnt this as a young teen, the day a poet pointed out that men are attracted towards
females’ outer beauty. Does it mean the not so beautiful people are not loved in this world? I think to
escape this situation an individual should invest their time in self-love. It’s the unconditional support and
compassion that translates to good health, great self-esteem, happiness, and well-being. Self-love allows us
to fill in the gaps external sources of love may leave. No one has a better understanding of our innermost
thoughts and needs than we do. Everyone is born with a unique set of characteristics so we should not
compare ourselves with others and let ourselves down But, it should not be confused with selfishness or
singing one’s own praises.

Footprints without Feet

a. Bholi was a weak and timid girl who was not loved and neither cared for. As a young
girl, she was ill-treated and never educated formally. Her face was pockmarked and she
was slow to understand things. Moreover, she was considered flawed and was engaged to
be married to a fifty-years old man for dowry. However, Bholi’s teacher shaped her into a
confident young girl who was aware of her rights and her needs. As a confident young
woman, Bholi refuses to marry when dowry is demanded and decides to remain a spinster all
her life. The changes in Bholi and the nurturing and love of her teacher make her stand for
herself and make a place in a conservative society and thrive in it.
b. It is essential for a person to live a contended life. One should believe in simple living and high
thinking. Not everyone is born with a golden spoon. However, with honesty and hard work,
we can always achieve our best. A person will never be happy if he keeps craving for what he
does not have. In this way, he may even ruin his present. Matilda had a loving and craving
for a life of luxury and wealth. She immersed herself in glamour when her husband got an
invitation to attend the ball. She did not care for her loving middle-class husband, rather she
was running after her fantasies. She always thought beyond her limits. She could have
gone to the minister’s party in the theatre dress, but she insisted on buying a new dress for 400
francs by shattering her husband’s dreams. Then she longed for jewellery which was
accessorial and unnecessary. She borrowed a diamond necklace and lost it. To repay its sum,
she had to sacrifice many years of her life and her husband’s income as well. Both lived in
utter poverty to repay it. Matilda lost all her charm and became pale and ugly, thus losing
her permanent beauty in search of momentary beauty. Her ostentation and vanity landed her
in trouble. Thus, it is apt to say that it is better to be satisfied with less rather showing off out
of one’s limits and Matilda is a good example of this.

c. The protagonist in the story Hari Singh is a young thief who has been caught stealing and
is being taken to prison. He narrates his story to a fellow passenger on the train and shares his
struggles and hardships that led him to a life of crime. The protagonist’s life had been marked
by poverty, hunger, and neglect, which forced him to take up stealing to survive. However,
during his conversation with the kind and understanding stranger, the thief begins to feel a sense of
empathy and compassion from the stranger. This sense of empathy and understanding is something
that the thief had never experienced before, and it touches him deeply. Anil listens to the thief’s
story with a sympathetic ear and tries to understand the reasons behind his actions.
A stranger’s kindness and compassion towards the thief creates a sense of trust and
understanding between them. This trust and understanding leads the thief to realize the
wrongness of his actions and he began to see the possibility of a better life.
The thief starts to feel a sense of remorse for his past deeds and decides to turn
over a new life. In the end, the thief decides to give up his life of crime and starts a fresh,
inspired by the kindness and trust shown to him by the stranger and he did not board the
train. Anil’s compassion and empathy for the thief led to his transformation from a hardened
criminal to a reformed person. In conclusion, the story shows that trust and compassion
can indeed reform a person. The stranger’s kindness and understanding towards the
thief created a sense of empathy and trust that ultimately led to the thief’s transformation.

d. Monday, 24 July, 1989 11:55pm

It's been a tough the past few years, for my wife, Matilda, and me. As I reflect on our experience, I can't
help but think about the broader societal issues that have contributed to our predicament.
It's clear to me now that our society is structured in a way that makes upward mobility difficult, if not
impossible. The rich get richer, while the rest of us struggle to make ends meet. We live in a world where
social status is determined by the amount of wealth one possesses, rather than by one's character or virtues.
This narrow view of success has led many people, including Matilda, to pursue material possessions at the
expense of their own happiness.
The loss of the borrowed necklace was a painful lesson for both of us. It reminded us that the pursuit of
social status and upward mobility can be a trap, leading people to sacrifice their happiness and well-being
in the pursuit of an unattainable dream.

e. The thief's character arc in the story shows a transformation from a career criminal to a person who
learns to trust and work hard for a living.
At the beginning of the story, the thief is portrayed as an experienced criminal who is focussed on
staying ahead of the law.
He is initially attracted to Anil because he believes he can exploit the young man's trusting nature.
As the story progresses, the thief begins to develop a friendship with Anil, who teaches him how to cook,
write and read.
This transformation of the thief's character reaches a climax when he is presented with an opportunity to
steal money from Anil, but instead chooses to resist the temptation and considers the consequences of his
actions.
This is a significant change from the beginning of the story when the thief was only interested in self-
gain.
Although the thief has not completely abandoned his criminal ways, the fact that he hesitated to jump on
the train to escape with the stolen money shows that he has developed a moral conscience.
The story implies that the thief has realized that a life of crime is not fulfilling and that he is now open to
exploring new opportunities for a better life.
[ It can also be observed that the thief struggles with his own moral compass throughout the narrative. At
first, he is solely focused on his own personal gain and uses his skills as a thief to achieve this. However, as
he begins working for Anil and starts to develop a relationship with him, he becomes conflicted about his
intentions to steal from him. Ultimately, he succumbs to his desire for money and steals from Anil, but he
also feels guilt and remorse afterwards. This internal conflict highlights the thief's character growth and
change throughout the story.]

f. Richard Ebright was a brilliant student. He had an urge to discover something new always. Albert
Einstein’s quote ‘The important thing is to never stop questioning’ suits Ebright perfectly. Since childhood,
Ebright was a curious child. He used to read a lot of books and research on several theories. When he lost
in his first entry at the county science fair, he didn’t stop trying. He learned that to win, he had to do a real
experiment and not just a display. Then he asked Dr. Urquhart about the ideas to work on ‘insects to win
the fair. Finally, he won the first prize in his second attempt. Later, he worked on Monarch pupa’s gold
spots and how cells read their DNA. He never satisfied himself after accomplishing his task. Nor did he
stop trying after failing at his attempt. He was determinant student who kept trying.

g. Mrs. Pumphrey was a rich woman who loved her dog very much. She loved to live a comfortable and
lavish life and also wanted her dog to spend a similar one. She had maintained a wardrobe full of fancy fur
coats, dresses, beds etc for Tricki. Apart from this, she used to overfeed Tricki out of her love and concern.
83
She used to serve him cod-liver oil and malt between the main meals and Horlicks after dinner to give him
strength. She never realised that Tricki was a greedy dog and this would spoil his health. She could not
even refuse to answer Tricks drooling for cream cakes and chocolates.

Her overfeeding worsened Tricki’s condition. This made the dog lazy, inactive and obese. He used to lie on
his rug and pant all day long. Mrs Pumphrey fed him excessively, spoiling Tricki’s health to such an extent
that he had to be hospitalised. Even in the hospital, she continued to convey Tricki her love through eggs,
wine, and brandy. Her fondness and care for Tricki proved that excess of everything made him fall sick.

h. He will never be able to believe any one. He would be further cautious about it. For the first time he let
his guards loose and he was trapped. He was chivalrous enough to believe a lady and take her on her face
value.

i. Fowler is a young romantic writer, who loves reading mysterious stories. He has a fantasy to meet a
secret agent or to see mysterious figures in the night, the crack of guns, drugs in wine, etc. He is not much
interested in meeting dull people. He gets easily bored and disappointed.
Though he is a writer, he is very timid by heart.

He easily loses his cool and gets nervous. The moment he enters Ausable’s room and his eyes meet Max
with an automatic pistol in his hand, he is thrilled. He is scared, the moment he realises that it is a waiter
and not the police who has knocked at the door. He wants to know about the man in the balcony. He has a
horrible experience with Ausable tackling risky situations so calmly.

j. Hari Singh, a boy of 15, was an experienced and successful thief. He was successful because of his
cleverness and intelligence. He planned everything meticulously before choosing his victims. He went to
places where he would meet an unsuspecting victim. He would then win his confidence to get a job. After
sometime he used to run away after stealing money from there.

Then-he used to change his name to be fool the police and his former employers. He was a liar. He got a
job as a cook, though he could not cook well. He was a greedy boy. He was cruel enough to rob a simple
and trusting man like Anil. He was able to steal six hundred rupees from his house. But, there is
transformation in the end of the story, when he decides to come back to Anil and keep his trust alive. This
shows that there is goodness concealed even in the worst of men.

k. The real culprit was the woman who pretended to be a member of the family living at Shotover Grange.
She tricked Horace into believing her, and cleverly took away all the jewels that were kept in the safe. .
Law of any land works on evidence. Horace Danby opened the safe without wearing his gloves.
He didn’t think it necessary to wear them as he thought he was working for ‘the lady of the house’. So
the police matched his fingerprints and he was arrested. The lady in the red was actually the real
culprit. She made Horace open the safe for her. She was also a thief and decamped with the jewels.
But in the eyes of the law, Horace was the real culprit because the evidence was against him.

l. Griffin was no doubt a brilliant scientist. The invisibility drug that he had made
proves this point. But as a human being he is a sadist who enjoys hurting others
besides being an unscrupulous criminal. He was a lawless person and lacked qualities
like compassion and kindness. His drug could have been a miracle but he used it
for unlawful activities. A true scientist makes discoveries for the benefit of society.
However, Griffin used his discovery to exploit others. Hence, Griffin may have
been a brilliant scientist but he was a failure as a human being.
m. Great and Mighty Think-Tank is introduced to us to be the most mighty, powerful and
intelligent creature. He is regarded as the ruler of Mars who is very proud of being Martian.
His pride makes him consult mirror to hear self-praise. He has a quick mind and wishes to
attack the Earth in a great hurry. The ways he makes guesses about the book are
also funny. He has respect for no one in front of him and regards others views as trifling.
But still his wit is too great. He makes Omega swallow the vitamin to decipher the code.
At every nursery rhyme, he prepares to go to Earth for invasion. He is also fickle minded.
One moment he orders the invasion to be taken next day and then he orders for immediate
invasion. He ordered the invasion flat to evacuate the entire planet and head for Alpha Centauri.
It is his commanding power which earns him name of general.

85

You might also like